100+ bonus english questions from act directions · 1 bonus questions to accompany the official act...

49
1 Bonus questions to accompany The Official ACT Prep Guide, 20162017 (ISBN 9781119225416), published by Wiley. 100+ BONUS ENGLISH QUESTIONS FROM ACT DIRECTIONS: In the passages that follow, certain words and phrases are underlined and numbered. Following each passage, you will find alternatives for the underlined part. In most cases, you are to choose the one that best expresses the idea, makes the statement appropriate for standard written English, or is worded most consistently with the style and tone of the passage as a whole. If you think the original version is best, choose "NO CHANGE." In some cases, you will find in a question about the underlined part. You are to choose the best answer to the question. You will also find questions about a section of the passage, or about the passage as a whole. These questions do not refer to an underlined portion of the passage, but rather are identified by a number in brackets. For each question, choose the alternative you consider best and fill in the corresponding oval on your answer document. Read each passage through once before you begin to answer the questions that accompany it. For many of the questions, you must read several sentences beyond the question to determine the answer. Be sure that you have read far enough ahead each time you choose an alternative. PASSAGE I: My "Sister" Ligia Every year my high school hosts international exchange students, those teenagers 1 join our senior class. Each student usually lives with the family of one of the seniors. I can recall students from Costa Rica, Italy, Norway, and Nigeria. Last year, one of our school's exchange students being 2 Ligia Antolinez, who 3 came from Bucaramanga, Colombia. I was a junior then 4 . I wasn't in any of Ligia's classes and didn't know her, but I saw her at school events, which are sometimes supported financially by local businesses. 5 About halfway through the school year, I learned that the exchange program was looking for a new home for Ligia. After a severe storm, the basement of her hosts 6 house had flooded, leaving two bedrooms unusable. The two "little brothers" of Ligia's host family, who had volunteered to move, to those bedrooms for a year 7 , had to be moved upstairs to the room Ligia was using. 8 I told my parents about Ligia's problem, which needed to be solved 9 . We agreed 10 that it would be fun to host a student from another country. My older sister had gotten married the summer before, so not only did we have a room for Ligia, and 11 we all admitted that the house had seemed too quiet lately. The second half of my junior year was anything but quiet. Introduced by me to my favorite music, at top volume, I started being taught by Ligia the most popular Colombian dance steps 12 . My father spoke fondly of the days before two teenagers taken 13 over the phone, the stereo, the kitchen-well, most of the house, really. My mother helped Ligia with her math homework, and Ligia taught Mom beginning Spanish. Both Ligia and I were studying French that year, and we practiced it at home. When we planned a surprise anniversary party for my mom and dad, we did it all right under their noses, in French. At the end of the year, Ligia had gone 14 home to Colombia. This year I'm busy with senior activities and with a part-time job. I'm trying to save enough to go see my new sister next year. [15]

Upload: others

Post on 20-Mar-2020

17 views

Category:

Documents


0 download

TRANSCRIPT

Page 1: 100+ BONUS ENGLISH QUESTIONS FROM ACT DIRECTIONS · 1 Bonus questions to accompany The Official ACT Prep Guide, 2016‐2017 (ISBN 9781119225416), published by Wiley. 100+ BONUS ENGLISH

Bonus questions to accompany The Official ACT Prep Guide, 2016‐2017 (ISBN 9781119225416), published by Wiley. 

100+ BONUS ENGLISH QUESTIONS FROM ACT DIRECTIONS: In the passages that follow, certain words and phrases are underlined and numbered. Following each passage, you will find alternatives for the underlined part. In most cases, you are to choose the one that best expresses the idea, makes the statement appropriate for standard written English, or is worded most consistently with the style and tone of the passage as a whole. If you think the original version is best, choose "NO CHANGE." In some cases, you will find in a question about the underlined part. You are to choose the best answer to the question. You will also find questions about a section of the passage, or about the passage as a whole. These questions do not refer to an underlined portion of the passage, but rather are identified by a number in brackets. For each question, choose the alternative you consider best and fill in the corresponding oval on your answer document. Read each passage through once before you begin to answer the questions that accompany it. For many of the questions, you must read several sentences beyond the question to determine the answer. Be sure that you have read far enough ahead each time you choose an alternative. PASSAGE I: My "Sister" Ligia Every year my high school hosts international exchange students, those teenagers 1 join our senior class. Each student usually lives with the family of one of the seniors. I can recall students from Costa Rica, Italy, Norway, and Nigeria. Last year, one of our school's exchange students being 2 Ligia Antolinez, who 3 came from Bucaramanga, Colombia. I was a junior then 4. I wasn't in any of Ligia's classes and didn't know her, but I saw her at school events, which are sometimes supported financially by local businesses. 5 About halfway through the school year, I learned that the exchange program was looking for a new home for Ligia. After a severe storm, the basement of her hosts 6 house had flooded, leaving two bedrooms unusable. The two "little brothers" of Ligia's host family, who had volunteered to move, to those bedrooms for a year 7, had to be moved upstairs to the room Ligia was using. 8 I told my parents about Ligia's problem, which needed to be solved 9. We agreed 10 that it would be fun to host a student from another country. My older sister had gotten married the summer before, so not only did we have a room for Ligia, and 11 we all admitted that the house had seemed too quiet lately. The second half of my junior year was anything but quiet. Introduced by me to my favorite music, at top volume, I started being taught by Ligia the most popular Colombian dance steps 12. My father spoke fondly of the days before two teenagers taken 13 over the phone, the stereo, the kitchen-well, most of the house, really. My mother helped Ligia with her math homework, and Ligia taught Mom beginning Spanish. Both Ligia and I were studying French that year, and we practiced it at home. When we planned a surprise anniversary party for my mom and dad, we did it all right under their noses, in French. At the end of the year, Ligia had gone 14 home to Colombia. This year I'm busy with senior activities and with a part-time job. I'm trying to save enough to go see my new sister next year. [15]

Page 2: 100+ BONUS ENGLISH QUESTIONS FROM ACT DIRECTIONS · 1 Bonus questions to accompany The Official ACT Prep Guide, 2016‐2017 (ISBN 9781119225416), published by Wiley. 100+ BONUS ENGLISH

Bonus questions to accompany The Official ACT Prep Guide, 2016‐2017 (ISBN 9781119225416), published by Wiley. 

1. A. NO CHANGE B. students, he or she is invited to C. students who D. students they 2. F. NO CHANGE G. students was H. students, named J. students, 3. A. NO CHANGE B. whom C. which D. she who 4. F. NO CHANGE G. junior, therefore, so H. junior because J. junior, since 5. Given that all of the choices are true, which one provides the most relevant information with regard to the narrator's familiarity with Ligia? A. NO CHANGE B. had read a story about her in our school paper, which is written by students interested in journalism. C. saw her at school events and had read a story about her in our school paper. D. had read a story about her when I was checking our school paper for local movie listings. 6. F. NO CHANGE G. her hosts' H. Ligia's hosts J. Ligias hosts' 7. A. NO CHANGE B. volunteered to move to those bedrooms for a year C. volunteered to move to those bedrooms for a year, D. volunteered, to move to those bedrooms for a year, 8. F. NO CHANGE G. upstairs to the room Ligia was using, which had been freshly painted just that year. H. upstairs (it was a two-story house) to Ligia's room. J. OMIT the underlined portion and end the sentence with a period.

Page 3: 100+ BONUS ENGLISH QUESTIONS FROM ACT DIRECTIONS · 1 Bonus questions to accompany The Official ACT Prep Guide, 2016‐2017 (ISBN 9781119225416), published by Wiley. 100+ BONUS ENGLISH

Bonus questions to accompany The Official ACT Prep Guide, 2016‐2017 (ISBN 9781119225416), published by Wiley. 

9. A. NO CHANGE B. problem, which was a dilemma. C. problem that needed a solution. D. problem. 10. Three of these choices indicate that the family felt confident about inviting Ligia to live in their home. Which choice does NOT do so? F. NO CHANGE G. decided H. knew J. supposed 11. A. NO CHANGE B. but C. while D. yet 12. F. NO CHANGE G. Introducing Ligia to my favorite music, at top volume, she started teaching me the most popular Colombian dance steps. H. Teaching me the most popular Colombian dance steps, Ligia was introduced by me to my favorite music, at top volume. J. I introduced Ligia to my favorite music , at top volume, and she started teaching me the most popular Colombian dance steps. 13. A. NO CHANGE B. took C. had took D. begun to take 14. F. NO CHANGE G. will have gone H. went J. goes 15. Which of the following true sentences, if inserted here, would best conclude the essay as well as maintain the positive tone established earlier in the essay? A. I'm afraid of flying, but I think I'll .be OK. B. I'm eager to eventually join the workforce fulltime. C. I've been practicing my Spanish,--and my dance steps. D. Senior activities are a lot of fun.

Page 4: 100+ BONUS ENGLISH QUESTIONS FROM ACT DIRECTIONS · 1 Bonus questions to accompany The Official ACT Prep Guide, 2016‐2017 (ISBN 9781119225416), published by Wiley. 100+ BONUS ENGLISH

Bonus questions to accompany The Official ACT Prep Guide, 2016‐2017 (ISBN 9781119225416), published by Wiley. 

PASSAGE II: Down at the Laundromat [1] Down the street from the college, I attend, 16 the Save-U Laundromat is always open, and someone is always there. [2] It was 17 on a corner, across the street; from 18 a drugstore on one side and a big park on the other. [3] The park isn't really a park at all but part of the grounds of a private boarding school. [4] But no one is ever around to enforce the threats, and in the summer everyone enjoys the benches, the grass, and the coolly magnificence 19 of the shade trees. [5] Signs are posted all over the lawn threatening every sort of drastic action against trespassers who wrongfully enter the property. 20 [21] The Save-U has a neon sign out front that says "Friendly 24-Hour Service," but as far as I can tell, no one really works there. The washers and dryers are lime green, and the paneling on the walls has been 22 painted to match, although it was later varnished with some kind of artificial wood grain finish. [23] I often stare at that paneling when I don't have a magazine or newspaper to read and don't want to do my schoolwork. Deep in thought, I contemplate the competence of the laundromat's interior designer. Some machines even provide a certain amount of sustenance and entertainment. 24 This laundromat has three soda machines, two candy machines, two pinball machines, five video machines, and a machine that eats dollar bills and spits out too much or too few 25 quarters. There are many regular customers whose faces have become familiar-mostly older people from around the neighborhood. [26] Usually a crowd of thirteen-year-old kids that 27 is gathered around the video machines, regardless of the time of day. Imagining all these people, it is that I know they 28 remain there even after I have left. I know that I could go in there anytime, and someone would look up from playing pinball or folding clothes and nods and smiles 29 at me. It is comforting to know that the Save-U Laundromat. And its 30 people are always nearby. 16. F. NO CHANGE G. college, I attend H. college I attend, J. college I attend 17. A. NO CHANGE B. is C. had been D. was located 18. F. NO CHANGE G. street·from, H. street, from J. street from

Page 5: 100+ BONUS ENGLISH QUESTIONS FROM ACT DIRECTIONS · 1 Bonus questions to accompany The Official ACT Prep Guide, 2016‐2017 (ISBN 9781119225416), published by Wiley. 100+ BONUS ENGLISH

Bonus questions to accompany The Official ACT Prep Guide, 2016‐2017 (ISBN 9781119225416), published by Wiley. 

19. A. NO CHANGE B. cool magnificence C. magnificently cool D. cool magnificent 20. F. NO CHANGE G. those who trespass by walking on private property. H. trespassers who ignore the signs and walk on the grass. J. trespassers. 21. For the sake of logic and coherence, Sentence 5 should be placed: A. where it is now. B. before Sentence 1. C. after Sentence 1. D. after Sentence 3. 22. F. NO CHANGE G. have been H. were J. are 23. At this point, the writer wants to add a sentence that would further describe the laundromat's paneling. Which of the following sentences would best accomplish this? A. I guess the brush strokes are intended to resemble wood grain, but they don't. B. I know that the varnish provides some protection for the wood paneling. C. To me, it seems that lime green was a bizarre choice for an interior wall paint. D. I imagine that the person who chose that color scheme must be a unique individual. 24. Which choice most effectively guides the reader from the preceding paragraph into this new paragraph? F. NOCHANGE G. The Save-U has to have friendly service because it is across the street from a park. H. Maybe what the Save-U means by friendly service is an abundance of machines. J. Washing machines are the Save-U's version of 24·hour service. 25. A. NO CHANGE B. many or too fewer C. many or too few D. much or few

Page 6: 100+ BONUS ENGLISH QUESTIONS FROM ACT DIRECTIONS · 1 Bonus questions to accompany The Official ACT Prep Guide, 2016‐2017 (ISBN 9781119225416), published by Wiley. 100+ BONUS ENGLISH

Bonus questions to accompany The Official ACT Prep Guide, 2016‐2017 (ISBN 9781119225416), published by Wiley. 

26. The writer is considering deleting the following phrase from the preceding sentence: -mostly older people from around the neighborhood. If the writer were to make this deletion, the essay would primarily lose: F. specific descriptive material. G. detail providing a logical transition. H. foreshadowing of the conclusion. J. an understatement of important information. 27. A. NO CHANGE B. kids who C. kids, and they D. kids 28. F. NO CHANGE G. It being that I imagine all these people, they H. Imagining all these people, they J. I imagine that all these people 29. A. NO CHANGE B. nod and smile C. nodding and smiling D. nods to smile 30. F. NO CHANGE G. Laundromat. Its H. Laundromat and that its J. Laundromat and its PASSAGE III: Bill Williams Brings America Home to Dinner

The following paragraphs may or may not be in the most logical order. Each paragraph is numbered in brackets, and question 45 will ask you to choose where Paragraph 1 should most logically be placed.

[1] You have to admire the honesty of a company who's 31 slogan is "Just About the Best." Glory Foods' president, and founder Bill Williams, 32 explains the unusual slogan by admitting that while he knows that his foods can't beat the taste of real home cooking, it does 33 come very close.

[2] Even as a child, Williams loved to prepare food, and as a young adult, he refined his cooking skills 34 at the prestigiously acclaimed 35 Culinary Institute of America. In 1989, he came up with his idea for a line of Southern-inspired cuisine, 36 a time when there were no convenience foods designed for African American consumers. Over the next three years, he developed a line of products that included canned greens, sweet potatoes, beans, and okra, as well as bottled hot sauce and cornbread mixes.

Page 7: 100+ BONUS ENGLISH QUESTIONS FROM ACT DIRECTIONS · 1 Bonus questions to accompany The Official ACT Prep Guide, 2016‐2017 (ISBN 9781119225416), published by Wiley. 100+ BONUS ENGLISH

Bonus questions to accompany The Official ACT Prep Guide, 2016‐2017 (ISBN 9781119225416), published by Wiley. 

[3] Eventually, Williams was ready to launch his products in grocery stores. Initially, Glory Foods were 37 first offered for sale in Ohio in 1992 and soon became available in neighboring states. Within a year, sales were twice the original projections. [38]

[4] The company's African American focus is evident in all aspects of Glory Foods. The firm's headquarters are located in the same black neighborhood where Williams grew up, and the company helps to support several local community projects. The firm also employs African American professional advisers 39 and subcontractors whenever possible and contracts African American farmers to grow much of the produce that goes into Glory Foods. [40]

[5] The company's name reflects this African American focus as well. Glory is meant to evoke 41 both the exultant spirit of gospel churches and the movie during the Civil War 42 of the same name, which tells the story of a black regiment.

[43]

[6] With twenty full-time employees in its administrative offices, Glory Foods has come a long way from its beginnings. America's dinner tables were 44 the beneficiaries of Bill Williams's drive, determination, and culinary expertise. 31. A. NO CHANGE B. whose C. that's D. that the 32. F. NO CHANGE G. president, and founder Bit! Williams H. president and founder Bill Williams, J. president and founder, Bill Williams, 33. A. NO CHANGE B. it has C. they do D. and that they 34. F. NO CHANGE G. his cooking skills were refined H. his skill in cooking was refined J. the refinement of his cooking skills occurred 35. A. NO CHANGE B. famed, renowned, and notable C. luscious· D. prestigious

Page 8: 100+ BONUS ENGLISH QUESTIONS FROM ACT DIRECTIONS · 1 Bonus questions to accompany The Official ACT Prep Guide, 2016‐2017 (ISBN 9781119225416), published by Wiley. 100+ BONUS ENGLISH

Bonus questions to accompany The Official ACT Prep Guide, 2016‐2017 (ISBN 9781119225416), published by Wiley. 

36. F. NO CHANGE G. He came up with his idea for a line of Southern-inspired cuisine in 1989, H. He came up in 1989, with his idea for a line of Southern-inspired cuisine, J. The idea came to him in 1989, that a line of Southern-inspired cuisine should be marketed, 37. A. NO CHANGE B. Glory Foods were C. They were originally D. At the outset, the earliest Glory Foods were 38. Given that all of the following sentences are true, which one would most effectively conclude this paragraph? F. Bill Williams's company continues to refine the recipes of its products. G. By 1995, Glory Foods were being distributed in twenty-two states. H. Today, there are several other companies that target their products to African American consumers. J. Bill Williams, however, sought the advice of food marketing experts. 39. A. NO CHANGE B. professional, advisers, C. professional advisers, D. professional advisers; 40. The writer is considering deleting the phrases "whenever possible" and "much of" from the preceding sentence. If the writer were to delete these phrases, would the meaning of the sentence change? F. Yes, because without these phrases, the reader would think that all of the subcontractors and farmers were African Americans. G. Yes, because without these phrases, the reader would not know that the company made an attempt to employ African American contractors in the production of its goods. H. No, because these phrases are examples of wordiness, and they can easily be eliminated from the sentence. J. No, because although these phrases describe the subcontractors and the farmers and provide interesting detail, they are not essential to the meaning of the sentence. 41. A. NO CHANGE B. · at evoking C. in evoking of D. OMIT the underlined portion. 42. The best placement for the underlined portion would be: F. where it is now. G. after the word name (but before the comma). H. after the word story. J. after the word regiment (ending the sentence with a period).

Page 9: 100+ BONUS ENGLISH QUESTIONS FROM ACT DIRECTIONS · 1 Bonus questions to accompany The Official ACT Prep Guide, 2016‐2017 (ISBN 9781119225416), published by Wiley. 100+ BONUS ENGLISH

Bonus questions to accompany The Official ACT Prep Guide, 2016‐2017 (ISBN 9781119225416), published by Wiley. 

43. At this point, the writer is considering adding the following sentence: The actor Denzel Washington starred in the film, which earned several awards. Should the writer make this addition? A. Yes, because the additional detail explains why the film Glory was so inspiring. B. Yes, because if readers understand that the film Glory earned awards, they will also understand why the company was named "Glory Foods." C. No, because the information distracts the reader from the focus of the essay. D. No, because the essay does not say if Bill Williams had ever met the actor Denzel Washington. 44. F. NO CHANGE G. had been H. would have been J. are

Question 45 asks about the preceding passage as a whole.

45. For the sake of logic and coherence, Paragraph 1 should be placed: A. where it is now. B. after Paragraph 2. C. after Paragraph 3. D. after Paragraph 6. PASSAGE IV: Pinball and Chance

[1]

Doesn't anyone play pinball anymore? I was disappointed the other day when I took my kids to a game arcade. Afterwards, I went to the movies. 46 Not one of the many colorful machines with flashing lights were a 47 pinball machine. Video games filled the room.

[2] [1] I can understand why video games might seem more attractive than pinball. [2] Video screens which have been

48 populated by movie stars, monsters, and heroes. [3] You can blow up cities, escape from dungeons, and battle all sorts of villains. [4] Pinball machines, on the other hand, are essentially all the same. [5] Some machines are bigger and fancier than others, but the object of pinball never changes: you have to keep a steel ball in play long enough to rack up a high score and win a free game. [49]

[3] The attractions of video games, however, are superficial and short-lived. As you guide your character through the game's challenges, you come to know exactly how the machine that's built to last 50 will respond to your every move. He or she learns 51 where the hazards lurk and the special weapons are hidden. Pinball, though, 52 can't be predicted with such accuracy. You never know when the ball will drain straight down the middle, out of reach of both flippers, Then again, you can sometimes get lucky, and a ball you thought was lost, will 53 inexplicably bounce back into play.

[4] It is the element of chance that makes pinball more interesting than video games. 54 Most video games are designed so that your main opponent in these video games 55 is a predictable computer program. Once you have mastered a game, the challenge is gone, and you must look 56 for a new game to conquer. After you learn the new game, you get bored again. The cycle keeps repeating. But in pinball, you have three factors to consider: you, the machine, and

Page 10: 100+ BONUS ENGLISH QUESTIONS FROM ACT DIRECTIONS · 1 Bonus questions to accompany The Official ACT Prep Guide, 2016‐2017 (ISBN 9781119225416), published by Wiley. 100+ BONUS ENGLISH

10 

Bonus questions to accompany The Official ACT Prep Guide, 2016‐2017 (ISBN 9781119225416), published by Wiley. 

chance, which is sometimes your enemy 57 sometimes your ally. No matter how many games you play on any pinball machine, the various times of each 58 game is different. That's what makes pinball a continually 59 challenge. 46. F. NO CHANGE G. I made my way to the movie theater after that. H. (The movie theater was my next stop.) J. OMIT the underlined portion. 47. A. NO CHANGE B. was a C. were an actual D. would have been an actual 48. F. NO CHANGE G. that are H. are J. OMIT the underlined portion. 49. For the sake of the logic and coherence of Paragraph 2, Sentence 4 should be: A. placed where it is now. B. placed after Sentence 1. C. placed after Sentence 5. D. OMITTED, because the paragraph focuses only on video games. 50. F. NO CHANGE G. machine, which is constructed durably, H. machine, which is built to last, J. machine 51. A. NO CHANGE B. We learn C. You learn D. People learned 52. Which of the following alternatives to the underlined portion would be LEAST acceptable? F. therefore, G. however, H. by contrast, J. on the contrary,

Page 11: 100+ BONUS ENGLISH QUESTIONS FROM ACT DIRECTIONS · 1 Bonus questions to accompany The Official ACT Prep Guide, 2016‐2017 (ISBN 9781119225416), published by Wiley. 100+ BONUS ENGLISH

11 

Bonus questions to accompany The Official ACT Prep Guide, 2016‐2017 (ISBN 9781119225416), published by Wiley. 

53. A. NO CHANGE B. lost will C. lost, will, D. lost will, 54. Which choice would most effectively and appropriately lead the reader from the topic of Paragraph 3 to that of Paragraph 4? F. NO CHANGE G. Pinball does share certain similarities with video games. H. Pinball, although less challenging than video games, can still be fun to play. J. Video games do generally evolve into subsequent editions or enhanced versions. 55. A. NO CHANGE B. during these video games C. in video games D. OMIT the underlined portion. 56. F. NO CHANGE G. you then looked H. one then looks J. one must look 57. A. NO CHANGE B. enemy, C. enemy; D. enemy, and, 58. F. NO CHANGE G. each H. each single unique J. every single time, each 59. A. NO CHANGE B. continuously C. continual D. continue

Page 12: 100+ BONUS ENGLISH QUESTIONS FROM ACT DIRECTIONS · 1 Bonus questions to accompany The Official ACT Prep Guide, 2016‐2017 (ISBN 9781119225416), published by Wiley. 100+ BONUS ENGLISH

12 

Bonus questions to accompany The Official ACT Prep Guide, 2016‐2017 (ISBN 9781119225416), published by Wiley. 

Question 60 asks about the preceding passage as a whole.

60. Suppose the writer had chosen to write an essay that indicates that pinball is superior to video games. Would this essay fulfill the writer's goal? F. No, because the writer admits that video games have become more popular than pinball machines. G. No, because the writer states that video games are designed to challenge the skills of the player. H. Yes, because the writer claims that pinball games require luck and are more visually attractive than video games. J. Yes, because the writer suggests that it is more difficult to become skilled at a pinball machine than at a video game. PASSAGE V: When a Computer Gets Sick ...

[1]

Imagine sitting in front of a computer monitor, filling the screen with your mind's jumbled thoughts. Tomorrow's assignment is slowly materializing before your eyes. Suddenly, without warning, each of the letters, in front of you tumbles 61 to the bottom of the screen. Is this a bad dream? Not exactly. The computer is probably sick, unless 62 the diagnosis may be that the computer has a virus.

[2] Analogous to a biological virus that takes over a living cell, a computer virus is a program, or set of instructions, that invades a computer either to create mischief or do real damage. The type of computer virus mentioned above is more mischievous than harmful. Eventually, the-letters reorder themselves on the screen. Not all viruses however, 63 straighten themselves out.

[3] Computer viruses range from being temporary annoyances to permanently destroying data. 64 Computer vandals rig these viruses to go off at a preset time. These bombs can permanently destroy data, and that can be disastrous to the operation 65 of a computer.

[4] Detection programs 66 are available that searches for and then destroys 67 computer viruses. Evidence that some software writers have played up the medical analogy being 68 found in the names of their programs: 69 Vaccine, Checkup, Antitoxin, and Disinfectant.

[5] As with 70 all diseases, the best cure is prevention. Experts suggest that you avoid borrowing computer disks because they might contain viruses. They warn that many of these viruses are quite sophisticated in their programming. 71 They also say that you should make copies of your computer files, so that if a virus does strike and you must delete your infected files, you will at least have backup copies. Experts also point out that using the Internet and World Wide Web has led to new risks of infection in the form of viruses hidden in programs downloaded, or copied, from these resources.

[6] If there is a virus in your system, you had hope that it better 72 responds to the appropriate treatment and therapy. Otherwise, you could be in for a long night at the computer.

Page 13: 100+ BONUS ENGLISH QUESTIONS FROM ACT DIRECTIONS · 1 Bonus questions to accompany The Official ACT Prep Guide, 2016‐2017 (ISBN 9781119225416), published by Wiley. 100+ BONUS ENGLISH

13 

Bonus questions to accompany The Official ACT Prep Guide, 2016‐2017 (ISBN 9781119225416), published by Wiley. 

61. A. NO CHANGE B. letters in front of you tumbles, C. letters in front of you, tumbles D. letters in front of you tumbles 62. F. NO CHANGE G. except H. and J. as if 63. A. NO CHANGE B. viruses; however, C. viruses, however D. viruses, however, 64. Which choice is the most effective first sentence of Paragraph 3? F. NO CHANGE G. Among the more serious viruses are those referred to as "bombs." H. Most people would agree that they'd rather have a computer virus than a virus that puts them in bed for a week. J. Despite technological advances, computers are still fragile devices in many ways. 65. A. NO CHANGE B. a devastative disaster to the operation C. devastation to the operating D. possibly disastrous to operating 66. F. NO CHANGE G. Detection programs that detect computer viruses H. Computer viruses can be found by detection programs that J. Detection programs that find computer viruses 67. A. NO CHANGE B. searches for and destroys C. search for and destroys D. search for and destroy 68. F. NO CHANGE G. analogy is H. analogy, having been J. analogy,

Page 14: 100+ BONUS ENGLISH QUESTIONS FROM ACT DIRECTIONS · 1 Bonus questions to accompany The Official ACT Prep Guide, 2016‐2017 (ISBN 9781119225416), published by Wiley. 100+ BONUS ENGLISH

14 

Bonus questions to accompany The Official ACT Prep Guide, 2016‐2017 (ISBN 9781119225416), published by Wiley. 

69. A. NO CHANGE B. programs; C. programs D. programs, 70. F. NO CHANGE G. Similarly to H. In the same way as J. According with 71. In this paragraph, the writer intends to recommend a number of specific ways to protect computer data against viruses. This is to be the second recommendation. Given that all of the choices are true, which one would best accomplish the writer's intention? A. NO CHANGE B. propose adding software that checks the spelling in the papers you write on your computer. C. advise you to give your system frequent checkups with antivirus programs. D. suggest that in order to protect your computer, you must be aware of the various ways to prevent viruses. 72. The best placement for the underlined portion would be: F. where it is now. G. after the word your. H. after the word had. J. after the word responds.

Questions 73-75 ask about the preceding passage as a whole.

73. Upon reviewing this essay and realizing that some information has been left out, the writer composes the following sentence, incorporating that information: Names like these suggest that the problem is serious. The most logical and effective place to add this sentence would be after the last sentence of Paragraph: A. 2. B. 3. C. 4. D. 5. 74. Paragraphs 1, 5, and 6 of this essay are written in the second person (you, your). If these paragraphs were revised so that the second-person pronouns were replaced with the pronouns one and one’s, the essay would primarily: F. gain a more polite and formal tone appropriate to the purpose of the essay. G. gain accessibility by speaking to a broader and more inclusive audience. H. lose the sense of directly addressing and advising the reader. J. lose the immediacy of its setting in terms of time and place.

Page 15: 100+ BONUS ENGLISH QUESTIONS FROM ACT DIRECTIONS · 1 Bonus questions to accompany The Official ACT Prep Guide, 2016‐2017 (ISBN 9781119225416), published by Wiley. 100+ BONUS ENGLISH

15 

Bonus questions to accompany The Official ACT Prep Guide, 2016‐2017 (ISBN 9781119225416), published by Wiley. 

75. Suppose the writer had decided to write an essay discussing the moral and ethical consequences of programming a computer virus to tamper with a computer system. Would this essay successfully fulfill the writer's goal? A. Yes, because the essay explains the moral and ethical consequences when a virus enters a computer system. B. Yes, because the essay details the process of ridding a computer system of viruses, which helps the reader understand the consequences of programming computer viruses. C. No, because the essay does not explain how to program a virus, so the reader has no basis for, making a moral or ethical judgment. D. No, because the essay limits itself to describing computer viruses and the basic precautions to be taken against them. PASSAGE VI: Grandpa's Remote Control

[1] My grandfather is not known for embracing technological change. He still drives 76 his '59 Chevy Impala. (He says, 77 he can't imagine needing frivolous options like automatic transmission or power steering.) So, when he has went 78 to buy a new color television-owing to the knowledge that 79 his old black-and-white model had finally quit-and the salesperson tried to talk him into buying a model with a remote control, he resisted. He said that he had two good legs and was perfectly capable of getting out of his chair. [80]

[2] However, the salesperson was persistent and, appealing to Grandpa's TV-viewing habits, described the various functions on the remote. However, my grandpa 81 could punch in the time, and the channel 82 of his favorite daily news program, and the TV would turn on that program at the proper time. In the end, Grandpa did buy the remote, and it has since become something he uses all the time.

[3] Grandpa is intrigued by the various uses for that remote. He has confided in me that the volume control is perfect for turning up the sound whenever Grandma asks him to take out the garbage. For example, 83 he says, the button that mutes the sound lets him cut them 84 off in midsentence.

[4] Grandpa's favorite feature on the remote is the sleep function. This option automatically turns the TV off after a preset amount of time, which is very convenient when 85 he falls asleep while watching a show. For him, Grandpa says what he wants his TV doing, even when he sleeps, is to know a source of both pleasure and power. 86

[5] [1] As for the programming function, Grandpa not only uses it for the news but also for playing jokes on his youngest grandchildren. [2] Explaining to the unsuspecting child that he has a remote control implanted in his little finger, Grandpa points 87 his finger at the TV and, to the child's amazement, seemingly turns it on. [3] I suppose Grandpa hasn't learned all the possible uses of the remote control, but I don't doubt he will continue to discover new and creative ways of using it. 88, 89 76. A. NO CHANGE B. change he still drives C. change still driving, D. change, and still driving 77. F. NO CHANGE G. says H. says, that J. says, that,

Page 16: 100+ BONUS ENGLISH QUESTIONS FROM ACT DIRECTIONS · 1 Bonus questions to accompany The Official ACT Prep Guide, 2016‐2017 (ISBN 9781119225416), published by Wiley. 100+ BONUS ENGLISH

16 

Bonus questions to accompany The Official ACT Prep Guide, 2016‐2017 (ISBN 9781119225416), published by Wiley. 

78. A. NO CHANGE B. had went C. went D. goes 79. F. NO CHANGE G. due to the understandable fact that H. because J. so 80. Given that all are true, which of the following additions to the preceding sentence (replacing "chair.") would be most relevant? A. chair that was made of black leather. B. chair when he wanted to change the channel. C. chair by the south window in the family room. D. chair where he liked to sit. 81. F. NO CHANGE G. Additionally, Grandpa H. Conversely, my grandpa J. Grandpa 82. A. NO CHANGE B. time and, the channel, C. time and the channel D. time and the channel, 83. F. NO CHANGE G. To illustrate, H. On the one hand, J. On the other hand, 84. A. NO CHANGE B. advertisers C. it D. its function 85. F. NO CHANGE G. convenient, when H. convenient. When J. convenient; when

Page 17: 100+ BONUS ENGLISH QUESTIONS FROM ACT DIRECTIONS · 1 Bonus questions to accompany The Official ACT Prep Guide, 2016‐2017 (ISBN 9781119225416), published by Wiley. 100+ BONUS ENGLISH

17 

Bonus questions to accompany The Official ACT Prep Guide, 2016‐2017 (ISBN 9781119225416), published by Wiley. 

86. A. NO CHANGE B. Even when he sleeps, Grandpa says that to know his TV is doing what he wants is a source of both pleasure and power for him. C. Doing what he wants, even when he sleeps, is to know his TV is a source of both pleasure and power for him, Grandpa says. D. Grandpa says that to know his TV is doing what he wants, even when he sleeps, is a source of both pleasure and power for him. 87. F. NO CHANGE G. pointing H. having pointed J. Grandpa has pointed 88. Which of the choices would provide an ending most consistent with the essay as a whole? A. NO CHANGE B. and he probably won't bother learning them either. C. so the salesperson should explain how to interpret the 200-page manual. D. and Grandma gratefully acknowledges this. 89. Upon reviewing Paragraph 5 and realizing that some information has been left out, the writer composes the following sentence: He programs the TV to turn on at a time when a grandchild will be visiting. The most logical placement for this sentence would be: F. before Sentence 1. G. after Sentence 1. H. after Sentence 2. J. after Sentence 3. Question 90 asks about the preceding passage as a whole. 90. The writer is considering deleting the first sentence from Paragraph 3. If the writer removed this sentence, the essay would primarily lose: A. information about the intriguing uses of the remote. B. details supporting the fact that Grandpa liked using the remote. C. a humorous blend of descriptive details and relevant information. D. a transition from the first two paragraphs to the rest of the essay. PASSAGE VII: "Topping Out" the Gateway Arch During the early morning hours of October 28, 1965, engineers 91 stationed 630 feet above the ground made careful measurements for the days 92 work. The results indicated a problem that threatened 93 to postpone and delay 94 the topping-out ceremony marking the placement of the final section between the two freestanding legs of the St. Louis Gateway Arch. [95] Thirty-two years of planning and effort resulted in this moment. In 1933, attorney and civic leader 96 Luther Ely Smith envisioned a memorial that would recognize St. Louis's major role in the westward expansion of the United States. [97] Architect Eero Saarinen, who created 98 the design that symbolized the memorial's theme of St. Louis as the "Gateway to the West." Meanwhile, the 99 arch would have a stainless steel exterior and interior structural supports made of concrete. Both legs of the arch would be built simultaneously using triangular sections. Those at the base of each arch leg would be the largest, with the higher sections progressively smaller. After nearly three years of construction, the day had come to place the final section at the top of the arch and finish the project. But a problem had arisen. The engineers confirmed that the heat of the sun had caused the south leg of the arch to expand five inches. This small 'but critical deviation caused concern that the two legs and the final

Page 18: 100+ BONUS ENGLISH QUESTIONS FROM ACT DIRECTIONS · 1 Bonus questions to accompany The Official ACT Prep Guide, 2016‐2017 (ISBN 9781119225416), published by Wiley. 100+ BONUS ENGLISH

18 

Bonus questions to accompany The Official ACT Prep Guide, 2016‐2017 (ISBN 9781119225416), published by Wiley. 

section might not connect properly. The engineers called in local firefighters in the hope that spraying the leg with water to cool it would make it contract. 100 The firefighters, using 700 feet of hose, were able to reach as high as 550 feet on the south leg in they're attempt to reduce its 101 expansion. The plan worked. By late morning, the crowd cheered as, welded to the two legs of the arch, the final section was hoisted up. 102 Over three decades and more than thirty years 103 of planning and building had come to a conclusion, 104 and the tallest monument in the United States was now complete. 91. A. NO CHANGE B. 1965, and engineers C. 1965. Engineers D. 1965; engineers 92. F. NO CHANGE G. days' H. day's J. days's 93. A. NO CHANGE B. had been threatened C. will have threatened D. threatens 94. F. NO CHANGE G. to a later time H. by delaying J. OMIT the underlined portion. 95. The writer is considering deleting the following from the preceding sentence: marking the placement of the final section between the two freestanding legs of the St. Louis Gateway Arch If the writer were to delete this phrase, the essay would primarily lose: A. a minor detail in the essay's opening paragraph. B. an explanation of the term "topping-out ceremony." C. the writer's opinion about the significance of the topping-out ceremony. D. an indication of the topping-out ceremony's importance to the people of St. Louis. 96. F. NO CHANGE G. attorney, and civic leader H. attorney and civic leader, J. attorney, and civic leader, 97. If the writer were to delete the preceding sentence, the paragraph would primarily lose: A. an explanation of why St. Louis had a major role in the westward expansion of the United States. B. details about what Luther Ely Smith thought the memorial he envisioned should look like. C. background information about the history leading to the Gateway Arch. D. biographical information about Luther Ely Smith.

Page 19: 100+ BONUS ENGLISH QUESTIONS FROM ACT DIRECTIONS · 1 Bonus questions to accompany The Official ACT Prep Guide, 2016‐2017 (ISBN 9781119225416), published by Wiley. 100+ BONUS ENGLISH

19 

Bonus questions to accompany The Official ACT Prep Guide, 2016‐2017 (ISBN 9781119225416), published by Wiley. 

98. F. NO CHANGE G. Saarinen, creator of H. Saarinen created J. Saarinen creating 99. A. NO CHANGE B. Therefore, the C. However, the D. The 100. F. NO CHANGE G. reduce. H. decrease. J. compress. 101. A. NO CHANGE B. they're attempt to reduce it's C. their attempt to reduce its D. their attempt to reduce it's 102. F. NO CHANGE G. as the crowd cheered, the final section was hoisted up and welded to the two legs of the arch. H. as the crowd cheered, welded to the two legs of the arch, the final section was hoisted up. J. the final section was hoisted up as the crowd cheered and welded to the two legs of the arch. 103. A. NO CHANGE B. decades amounting to more than thirty years C. decades-over thirty years- D. decades 104. Which of the following alternatives to the underlined portion would be LEAST acceptable in terms of the context of this sentence? F. reached completion, G. come to a halt, H. come to an end, J. ended, Question 105 asks about the preceding passage as a whole. 105. Suppose the writer had intended to write a brief essay that describes the entire process of designing and building the St. Louis Gateway Arch. Would this essay successfully fulfill the writer's goal? A. Yes, because it offers such details as the materials used to make the exterior and the interior structural supports. B. Yes, because it explains in detail each step in the design and construction of the arch. C. No, because it focuses primarily on one point in the development of the arch rather than on the entire process. D. No, because it is primarily a historical essay about the early stages in the development of the arch.

Page 20: 100+ BONUS ENGLISH QUESTIONS FROM ACT DIRECTIONS · 1 Bonus questions to accompany The Official ACT Prep Guide, 2016‐2017 (ISBN 9781119225416), published by Wiley. 100+ BONUS ENGLISH

20 

Bonus questions to accompany The Official ACT Prep Guide, 2016‐2017 (ISBN 9781119225416), published by Wiley. 

100+ BONUS ENGLISH QUESTIONS: EXPLANATORY ANSWERS

PASSAGE I: My "Sister" Ligia

Question 1. The best answer is C because it appropriately uses the relative pronoun who to introduce the clause that modifies students —“who join our senior class." Besides introducing that clause, the pronoun who also functions as the subject of the clause.

The best answer is NOT:

A because it creates a comma splice (two or more complete sentences separated only by a comma). The phrase "those teenagers" is the subject of the second complete sentence.

B because it, too, produces a comma splice. In addition, it creates grammatical disagreement between the plural students and the singular he or she.

D because it creates a run-.on, or fused, sentence. There is no punctuation or conjunction (connecting word) between the two statements.

Question 2. The best answer is G because it provides the predicate was, which produces a complete sentence. Remember that a statement that has no predicate verb is a sentence fragment (an incomplete sentence).

The best answer is NOT:

F because it uses the verb form being, which is a participle. Because it is a verb form, a participle is often mistaken for the main verb in a sentence. This statement has no predicate, so it is a sentence fragment.

H because it has no predicate verb. Without a predicate, the statement is a sentence fragment and does not express a complete thought.

J because it lacks a verb and therefore creates another sentence fragment.

Question 3. The best answer is A because it correctly uses the pronoun who to introduce the clause that describes Ligia Antolinez. In this sentence, who is required because it refers to a person. The pronoun who is also appropriate because it functions as the subject of the clause.

The best answer is NOT:

B because it uses the object pronoun whom instead of the subject pronoun who.

C because it uses the pronoun which when the personal pronoun who is required. In general, who refers to people and which refers to objects, events, or animals.

D because it inserts an unnecessary pronoun, she. Because who is the subject of the descriptive clause, the pronoun she has no function in this sentence.

Question 4. The best answer is F because this short sentence expresses a complete thought and is clear, concise, and grammatically sound. It also logically fits between the preceding sentence and the sentence that follows.

The best answer is NOT:

Page 21: 100+ BONUS ENGLISH QUESTIONS FROM ACT DIRECTIONS · 1 Bonus questions to accompany The Official ACT Prep Guide, 2016‐2017 (ISBN 9781119225416), published by Wiley. 100+ BONUS ENGLISH

21 

Bonus questions to accompany The Official ACT Prep Guide, 2016‐2017 (ISBN 9781119225416), published by Wiley. 

G because it creates a statement that is not logical. The conjunction (connecting word) therefore suggests a cause-effect relationship that makes no sense. The fact that the narrator "was a junior then" was not the cause of her not being in classes with Ligia.

H because it makes no sense. It illogically suggests that the narrator was a junior because she "wasn't in any of Ligia's classes."

J because it creates the same error as H does by illogically suggesting that the narrator was a junior because she "wasn't in any of Ligia's classes."

Question 5. The best answer is C because it adds a relevant detail that fits with the point of the rest of the sentence. The narrator didn't know Ligia but knew of her. The narrator "saw her at school events and had read a story about her." Considering the other choices, C provides the most relevant information about the narrator's familiarity with Ligia.

The best answer is NOT:

A because the phrase "which are sometimes supported financially by local business" is not relevant with regard to the narrator's knowledge of Ligia.

B because the information that the school paper "is written by students interested in journalism" is irrelevant to the writer's purpose here.

D because information about the narrator checking the paper "for local movie listings" is a detail that distracts the reader from main point of the sentence.

Question 6. The best answer is G because the plural possessive form hosts' is the correct punctuation here. The phrase "her hosts' house" shows possession and requires an apostrophe.

The best answer is NOT:

F because it fails to use the required apostrophe to show possession.

H because again, it fails to use an apostrophe after the sin hosts.

J because although it does use the required apostrophe after hosts, it fails to use the required apostrophe to show possession in Ligia's.

Question 7. The best answer is C because it correctly inserts a comma after the word year.

Notice that this comma is necessary to set off the nonessential clause "who had volunteered to move." A nonessential clause adds information that is not necessary to the main idea. Nonessential clauses are set off with commas on both ends.

The best answer is NOT:

A because it inserts an unnecessary and confusing comma after move.

B because it fails to insert the required comma after year. This comma is necessary to set off the nonessential clause that begins with "who had volunteered ...."

D because it inserts an unnecessary and confusing comma after volunteered.

Question 8. The best answer is F because it provides the best explanation of the host family's situation and why Ligia needed a place to stay. This choice provides relevant information to show that after the storm, the two brothers needed their upstairs room back-the same room that

Ligia had been using.

Page 22: 100+ BONUS ENGLISH QUESTIONS FROM ACT DIRECTIONS · 1 Bonus questions to accompany The Official ACT Prep Guide, 2016‐2017 (ISBN 9781119225416), published by Wiley. 100+ BONUS ENGLISH

22 

Bonus questions to accompany The Official ACT Prep Guide, 2016‐2017 (ISBN 9781119225416), published by Wiley. 

The best answer is NOT:

G because it adds irrelevant information. The detail that the upstairs room "had been freshly painted" distracts the reader from the main point of the sentence, which is to show why Ligia needed another place to live.

H because the statement in parentheses, "it was a two-story house," is also irrelevant to the writer's purpose here.

J because if the sentence simply ended with "had to be moved," it would not clearly explain why Ligia needed a new place to live.

Question 9. The best answer is D because, of the four choices, D makes the point in the clearest, most concise way.

The best answer is NOT:

A because it is redundant; that is, it repeats an idea that has already been stated. The sentence states that the narrator was aware of "Ligia's problem." Adding that this problem "needed to be solved" is overstating the obvious. It is better to end the sentence with the word problem.

B because it, too, is unnecessarily wordy. The word problem already implies a dilemma.

C because it is incorrect in the same way that A and B are. That Ligia's problem "needed a solution" overstates the obvious and lacks conciseness.

Question 10. The best answer is J. You need to pay close attention to the stated question.

It asks you for the choice that does not show that the narrator's "family felt confident about inviting Ligia to live in their home." In other words, the question tells you that the best answer is the worst word choice with respect to the writer's purpose. The verb supposed is the only choice that does not show that "the family felt confident," so it is the best answer to this question.

The best answer is NOT:

F because it does indicate the family's confidence. Because the family agreed to host Ligia, they "felt confident about inviting Ligia to live in their home."

G because it, too, does indicate the family's confidence.

H because the word knew is appropriate in this context and, like F and G, does indicate confidence.

Question 11. The best answer is B because it correctly uses the correlative conjunctions not only and but. Correlative conjunctions connect similar ideas and are always used in pairs. In this

sentence, the pair is "not only did we have a room..., but... the house had seemed too quiet."

The conjunctions not only and but logically connect the two reasons that the family agreed to host Ligia.

The best answer is NOT:

A because "not only ... and" does not logically connect the two reasons, and it is not idiomatic (it does not conform to standard written English).

C because it is incorrect in the same way that A is. It creates a statement that is not logical.

D because it is incorrect in the same way that A and C are. It also fails to use proper correlative conjunctions.

Page 23: 100+ BONUS ENGLISH QUESTIONS FROM ACT DIRECTIONS · 1 Bonus questions to accompany The Official ACT Prep Guide, 2016‐2017 (ISBN 9781119225416), published by Wiley. 100+ BONUS ENGLISH

23 

Bonus questions to accompany The Official ACT Prep Guide, 2016‐2017 (ISBN 9781119225416), published by Wiley. 

Question 12. The best answer is J because it is the clearest and most logical, and it is the most structurally sound. The two clauses in this sentence are parallel and logically follow one another. The second clause, "she started teaching me ... dance steps," logically follows "I introduced Ligia to my favorite music."

The best answer is NOT:

F because using the passive voice ("I started being taught by Ligia") makes the sentence confusing. It is difficult for the reader to tell what the subject and object of this sentence are. The arrangement of the sentence elements is also confusing and garbled.

G because it has an incorrect modifier. When a modifying phrase containing a verbal comes at the beginning of a sentence, the phrase is followed by a comma. The word that the phrase modifies should immediately follow the comma. In this case, the modifying phrase "Introducing Ligia to my favorite music, at top volume," is followed by the pronoun she, instead of the pronoun I (to refer to the narrator).

H because the modifying word after the introductory phrase is correct, but the rest of the sentence is weak because it relies on the passive voice ("was introduced by me"). In addition, the phrase at the end of the sentence, "at top volume," is misplaced.

Question 13. The best answer is B because it uses the correct verb form. The entire essay is in the past tense, so the past tense took is required here.

The best answer is NOT:

A because it uses an incorrect verb form here-the past participle taken without an auxiliary verb (for example, had).

C because had took is an incorrect verb form.

D because it uses an incorrect verb form- the past participle begun without an auxiliary (helping) verb.

Question 14. The best answer is H because it appropriately uses the past tense verb form (went) to show that the event (Ligia's going home) occurred at a specific past time.

The best answer is NOT:

F because it inappropriately uses the past perfect tense. The perfect tenses are mainly used to show that one event happened before another event, which is not the case here.

G because it uses a future tense (in this case, the future perfect) to refer to a past event. You can tell that this is a past event by reading the sentence that follows.

J because it uses the present tense goes to refer to an event that happened in the past.

Question 15. The best answer is C because it concludes the essay by referring back to topics that were previously mentioned: that Ligia spoke Spanish and that she taught the narrator Colombian dance steps. In addition, it logically follows the preceding sentence by explaining how the narrator continues to make plans for a visit to Ligia.

The best answer is NOT:

A because it does somewhat follow the preceding sentence, but it does not refer back to any of the ideas mentioned in the essay. It is therefore a poor conclusion when compared with C.

Page 24: 100+ BONUS ENGLISH QUESTIONS FROM ACT DIRECTIONS · 1 Bonus questions to accompany The Official ACT Prep Guide, 2016‐2017 (ISBN 9781119225416), published by Wiley. 100+ BONUS ENGLISH

24 

Bonus questions to accompany The Official ACT Prep Guide, 2016‐2017 (ISBN 9781119225416), published by Wiley. 

B because this is a poor conclusion for the essay because it introduces an entirely new topic: joining the workforce.

D because although the essay does refer earlier to "senior activities," this is also a weak conclusion because it is a vague generalization. In addition, it does not logically follow the statement that the narrator is "trying to save enough to go see my new sister next year."

PASSAGE II: Down at the Laundromat

Question 16. The best answer is H because it uses a comma after attend to appropriately set off the introductory phrase from the main clause. Without this comma, the reader might be confused and think that the narrator attended the laundromat.

The best answer is NOT:

F because it adds an unnecessary and confusing comma between college and I.

G because it, too, adds an unnecessary and confusing comma between college and I. In addition, it fails to add the appropriate comma after attend.

J because, like G, it omits the comma after attend, producing a potentially confusing statement for readers.

Question 17. The best answer is B because it appropriately uses the present tense to describe an event that is happening in the present time. Notice that the writer begins the essay in the present tense ("the Save-D Laundromat is always open").

The best answer is NOT:

A because it makes a confusing tense shift from present (is) to past (was).

C because it makes another confusing tense shift-this time from the present tense to the past perfect tense.

D because, like A, it makes a confusing tense shift from present to past.

Question 18. The best answer is J because no punctuation is needed here. The absence of punctuation creates the clearest and most understandable sentence.

The best answer is NOT:

F because it places a semicolon between two descriptive phrases, which is a misuse of the semicolon.

G because it inserts an unnecessary and confusing comma between a preposition and its object.

H because it places an unnecessary comma between the two descriptive phrases. There is no pause or separation between the phrases "across the street" and "from a drugstore." They belong together as one description.

Question 19. The best answer is B because it is grammatically correct. In this sentence, cool is used as an adjective to modify the noun magnificence.

The best answer is NOT:

Page 25: 100+ BONUS ENGLISH QUESTIONS FROM ACT DIRECTIONS · 1 Bonus questions to accompany The Official ACT Prep Guide, 2016‐2017 (ISBN 9781119225416), published by Wiley. 100+ BONUS ENGLISH

25 

Bonus questions to accompany The Official ACT Prep Guide, 2016‐2017 (ISBN 9781119225416), published by Wiley. 

A because coolly is an adverb, and an adverb cannot be used to modify a noun. (Adverbs generally modify verbs or adjectives.)

C because it uses an adjective phrase-"magnificently cool"- where a noun is required. The complete phrase "The magnificently cool of the shade trees" is both ungrammatical and confusing.

D because it uses an adjective phrase-"cool magnificent"-where a noun is called for. The phrase "the cool magnificent of the shade trees" is ungrammatical.

Question 20. The best answer is J because it states the idea most clearly and concisely. It

does not repeat the same idea twice, and it does not add unnecessary words to the sentence.

The best answer is NOT:

F because it is redundant (repeats the same idea) and wordy (adds unnecessary words). The descriptive phrase "who wrongfully enter the property" is really a repetition of the same idea expressed by the use of the word trespassers. In other words, the descriptive phrase restates the obvious.

G because it has the same problem that F does. The phrase "who trespass by walking on private property" adds wordiness and redundancy.

H because it, too, is wordy. It is not necessary to state the obvious. It is already clear to readers that people" who ignore the signs and walk on the grass" are trespassers.

Question 21. The best answer is D because placing Sentence 5 after Sentence 3 makes the paragraph logical and coherent. If you read Sentences 3, 4, and 5 carefully, you will notice that Sentence 4 does not logically follow Sentence 3. The opening clause in Sentence 4, "But no one is ever around to enforce the threats," has no antecedent to connect it back to Sentence 3. The threats in Sentence 4 refer to the "signs ... posted all over the lawn" that are referred to in Sentence 5. Therefore, Sentence 4 makes the best sense when it follows Sentence 5 rather than precedes it.

The best answer is NOT:

A because leaving Sentence 5 where it is now is not logical for the reasons explained above.

B because Sentence 5 would be a poor and illogical introduction to this paragraph because the reader would not know to what lawn the writer was referring. In addition, the paragraph would make no sense if Sentence 1 followed Sentence 5.

C because this arrangement of the sentences is also illogical and would confuse the reader. Placing Sentence 5 after the description of the laundromat in Sentence 1 makes no sense because the signs on the lawn are on the grounds of a school and are not part of the laundromat.

Question 22. The best answer is F because the singular verb has agrees with the singular noun paneling. Remember that the verb must agree in number with its subject (in this case, paneling) and not the object of the preposition (in this case, the plural noun walls).

The best answer is NOT:

G because the plural verb have does not agree in number with the singular noun paneling.

Page 26: 100+ BONUS ENGLISH QUESTIONS FROM ACT DIRECTIONS · 1 Bonus questions to accompany The Official ACT Prep Guide, 2016‐2017 (ISBN 9781119225416), published by Wiley. 100+ BONUS ENGLISH

26 

Bonus questions to accompany The Official ACT Prep Guide, 2016‐2017 (ISBN 9781119225416), published by Wiley. 

H because, like G, it has an agreement problem. The plural verb were does not agree in number with the singular noun paneling.

J because, like G and H, it has an agreement problem. The plural verb are does not agree in number with the singular noun paneling.

Question 23. The best answer is A because it provides the added detail asked for in the question. Pay close attention to the stated question. It asks for the sentence that would best accomplish the writer's wish to "further describe the laundromat's paneling." A is the only choice that accomplishes this goal. It further describes the "artificial wood grain finish" by showing that it was intended to resemble wood grain but doesn't.

The best answer is NOT:

B because it does not provide a detail that further describes the paneling. Although B mentions the paneling, it does not offer a further description of it. Rather, it adds a detail that is irrelevant to the paragraph.

C because it, too, fails to further describe the paneling. Instead, it offers an opinion about the color of the paneling.

D because it is incorrect in the same way that C is. It offers an opinion about the person who "chose that color scheme," but it does not further the description of the paneling.

Question 24. The best answer is H because it effectively links the new paragraph to the question implied by the preceding paragraph: Why· does the neon sign promise friendly service? H also provides the most effective introduction to the information in the new paragraph.

The best answer is NOT:

F because it does not link the theme of friendly service that is questioned in the preceding paragraph to the description of the machines in this new paragraph. In addition, it shifts to a more formal tone.

G because it makes no sense. Being "across the street from a park" has nothing to do with friendly service. Besides, in the first paragraph, the writer states that "the park isn't really a park at all."

J because it misleads the reader into thinking that the topic of the new paragraph will be "washing machines."

Question 25. The best answer is C because it provides the correct adjectives (many, few) to describe the quarters. The phrase "too many or too few quarters" describes a relationship of number.

The best answer is NOT:

A because it is ungrammatical. It incorrectly uses an adjective of quantity (much) when an adjective of number (many) is required.

B because it incorrectly adds the modifier too to the comparative adjective fewer.

D because it is incorrect in the same way that A is. It incorrectly uses an adjective of quantity (much) when an adjective of number (many) is required.

Page 27: 100+ BONUS ENGLISH QUESTIONS FROM ACT DIRECTIONS · 1 Bonus questions to accompany The Official ACT Prep Guide, 2016‐2017 (ISBN 9781119225416), published by Wiley. 100+ BONUS ENGLISH

27 

Bonus questions to accompany The Official ACT Prep Guide, 2016‐2017 (ISBN 9781119225416), published by Wiley. 

Question 26. The best answer is F because the phrase "mostly older people from around the neighborhood" specifically describes the group of "regular customers" mentioned in the first part of the sentence. If the phrase were deleted, specific descriptive material would be lost.

The best answer is NOT:

G because the phrase is not a detail that provides a logical transition because the sentence that follows describes a different group of customers.

H because the phrase does not foreshadow the conclusion. The writer does not conclude the essay with "older people from around the neighborhood"; rather, the essay ends with all the people who frequent the laundromat.

J because this information is not understated. Also, it is not "important information" - essential to the essay - but, rather, an interesting and relevant side note.

Question 27. The best answer is D because it results in a complete sentence. The complete subject of the sentence is "a crowd of thirteen-year-old kids." The predicate is immediately follows this subject.

The best answer is NOT:

A because it creates an incomplete sentence. It improperly inserts the pronoun that between the subject and predicate, which results in a sentence fragment.

B because it is incorrect in the same way that A is. It inserts the relative pronoun who between the subject and the predicate and creates an incomplete sentence.

C because the use of the comma and the conjunction and generally indicates that the sentence contains two independent clauses, but in this case, there is only one independent clause. "Usually a crowd of thirteen-year old kids" is a phrase, not a clause, because it has no verb. Meanwhile, in the main clause, the predicate is is disagreeing in number with the subject they.

Question 28. The best answer is J because it is clear, concise, and structurally sound. It clearly expresses the idea that it is the writer who is imagining.

The best answer is NOT:

F because it has an ineffective sentence structure that results in a dangling modifier. When a modifying phrase containing a verbal comes at the beginning of a sentence, the phrase is followed by a comma ("Imagining all these people,"). Following the comma is the word that this phrase modifies. Notice in this sentence that the pronoun it incorrectly follows the introductory phrase. The modifying word should be the pronoun I.

G because it creates a confusing and unclear statement. In the clause "It being that I imagine all these people," the reader does not know to what the pronoun It refers.

H because it has a dangling modifier. It has a problem that is similar to the one in F. The pronoun I should follow the introductory clause, not the pronoun they.

Question 29. The best answer is B because it is grammatically correct, and the verbs in the sentence are parallel (maintain the same verb tense). The appropriate verbs here are "nod and smile" because they correctly follow the auxiliary (helping) verb would: "someone would look up ... and [would] nod and smile at me." Although the helping verb would is not repeated before "nod and smile," it is implied.

Page 28: 100+ BONUS ENGLISH QUESTIONS FROM ACT DIRECTIONS · 1 Bonus questions to accompany The Official ACT Prep Guide, 2016‐2017 (ISBN 9781119225416), published by Wiley. 100+ BONUS ENGLISH

28 

Bonus questions to accompany The Official ACT Prep Guide, 2016‐2017 (ISBN 9781119225416), published by Wiley. 

The best answer is NOT:

A because is ungrammatical. It incorrectly uses the third-person singular verb form ("nods and smiles") after the implied helping verb would.

C because the use of the present participle ("nodding and smiling") after the helping verb would is ungrammatical. Note also that that sentence lacks parallelism: "someone would look up ... and nodding and smiling at me." There is an illogical tense shift from the present tense ("would look") to the present participle ("nodding and smiling").

D because it makes the same mistake as A. In addition, it results in an illogical statement.

Question 30. The best answer is J because it is the only choice that is a complete sentence with appropriate sentence structure. Notice that this sentence has a compound subject ("the Save-U Laundromat" and "its people"). This construction makes the subject of the sentence clear.

The best answer is NOT:

F because it inserts incorrect punctuation (a period) that results in two sentence fragments (incomplete sentences).

G because it also inserts incorrect punctuation. The clause after the period does create a complete sentence, but the opening phrase "It is comforting to know that the Save-U Laundromat" is not a complete sentence; the relative clause does not contain a predicate.

H because it has a confusing and ineffective sentence structure. Inserting the relative pronoun that between the conjunction and and the pronoun its results in a sentence with faulty parallelism. The first of these two relative clauses ("that the Save-U Laundromat") is incomplete.

PASSAGE III: Bill Williams Brings America Home to Dinner

Question 31. The best answer is B because it correctly uses the relative pronoun whose to introduce the clause that describes the company that the narrator admires. The pronoun whose indicates possession and is appropriate here.

The best answer is NOT:

A because it uses a contraction (who's) instead of the required pronoun (whose). The contraction who's means "who is" and does not indicate possession.

C because it has a problem that is similar to the one in A. It incorrectly uses the contraction that's, which means "that is."

D because it is creates an unclear statement, and it fails to use the proper relative pronoun whose to indicate possession.

Question 32. The best answer is J because it provides the best punctuation to set off the appositive "Bill Williams." An appositive is a noun or pronoun that identifies and follows another noun or pronoun. In this sentence, "Bill Williams" identifies "Glory Foods' president and founder." Appositives are set off by commas (except when the apposition is restrictive, such as in the phrase "my sister Sue" when I have three sisters).

The best answer is NOT:

Page 29: 100+ BONUS ENGLISH QUESTIONS FROM ACT DIRECTIONS · 1 Bonus questions to accompany The Official ACT Prep Guide, 2016‐2017 (ISBN 9781119225416), published by Wiley. 100+ BONUS ENGLISH

29 

Bonus questions to accompany The Official ACT Prep Guide, 2016‐2017 (ISBN 9781119225416), published by Wiley. 

F because it inserts an unnecessary and confusing comma between president and and. In addition, it fails to set off the appositive with a necessary comma between founder and Bill.

G because, like F, it inserts an unnecessary and confusing comma between president and and.

H because it fails to set off the appositive by adding the necessary comma between founder and Bill.

Question 33. The best answer is C because the third-person plural pronoun they clearly refers back to the plural noun foods.

The best answer is NOT:

A because the singular pronoun it has no logical antecedent. An antecedent is the word or phrase to which a pronoun refers. In this sentence, the antecedent foods is plural and requires a plural pronoun (they).

B because it has the same problem described in A.

D because it creates faulty coordination and a confusing statement. The phrase "and that they" does not effectively coordinate with "that while he knows."

Question 34. The best answer is F because it provides the clearest, most concise statement, and it uses modifiers correctly. Note that the pronoun he directly follows and correctly modifies the adjective phrase "as a young adult."

The best answer is NOT:

G because it creates a dangling modifier. The phrase "as a young adult" does not logically refer to or modify the noun phrase it precedes ("his cooking skills"). This arrangement of sentence elements results in a confusing statement.

H because although the wording is somewhat different, the problem here, a dangling modifier, is the same as that in G. Here, "his skill" is not "a young adult."

J because the problem is much like that in both G and H. In this statement, "the refinement of his cooking skills" is not "a young adult."

Question 35. The best answer is D because it is the dearest and most concise statement.

The writer logically describes the Culinary Institute of America as "prestigious."

The best answer is NOT:

A because it is redundant. That is, it repeats the same idea twice: prestigious and acclaimed mean the same thing.

B because it, too, is redundant. It repeats the same idea three times: famed, renowned, and notable all have similar meanings.

C because the adjective luscious makes no sense in this context. Food might be "luscious," but an institute would not be.

Question 36. The best answer is G because the sentence parts are arranged in a logical order so that they modify the appropriate elements. This results in the clearest word order for this sentence.

The best answer is NOT:

Page 30: 100+ BONUS ENGLISH QUESTIONS FROM ACT DIRECTIONS · 1 Bonus questions to accompany The Official ACT Prep Guide, 2016‐2017 (ISBN 9781119225416), published by Wiley. 100+ BONUS ENGLISH

30 

Bonus questions to accompany The Official ACT Prep Guide, 2016‐2017 (ISBN 9781119225416), published by Wiley. 

F because the clauses are put together in a way that confuses the reader. The noun phrase "a line of Southern-inspired cuisine" doesn't connect logically with the noun and the clause that immediately follows it - "a time when there were no convenience foods designed for African American consumers."

H because it is ambiguous. It is unclear what is meant by the opening clause "He came up in 1989."

J because, like F, it strings clauses together in a confusing way.

Question 37. The best answer is B because it creates the clearest, most logical, and most concise statement. This is another case (like question 35) where the least wordy choice is best.

The best answer is NOT:

A because it is redundant. It repeats the same idea twice. The introductory word Initially is redundant because the sentence later states that "Glory Foods were first offered for sale in Ohio in 1992."

C because it is redundant. In this case, the words originally and first mean the same thing.

D because it is both wordy and redundant. The phrases "At the outset" and "the earliest" both imply the same thing.

Question 38. The best answer is G because it most effectively concludes this paragraph by continuing the theme of Glory Foods' business success. That Glory Foods "were being distributed in twenty-two states" logically follows the information that "sales were twice the original projections."

The best answer is NOT:

F because it changes the topic by discussing recipes instead of the company's success.

H because it shifts to an entirely new topic, that of "several other companies."

J because the use of the word however makes this statement illogical. As it is used here, however indicates that this sentence is going to contradict the statement in the preceding sentence, but this sentence does not do that.

Question 39. The best answer is A because no punctuation is needed here. The absence of commas makes this the dearest sentence.

The best answer is NOT:

B because it adds unnecessary commas and incorrectly treats " professional, advisors, and subcontractors" as if they were items in a series, but professional functions as an adjective modifying the noun advisors.

C because the unnecessary comma between the two parts of the compound direct object "advisors and subcontractors" adds confusion to the sentence.

D because it inserts an inappropriate and confusing semicolon between advisors and and.

Question 40. The best answer is F because without the qualifying phrases the sentence would give the impression that all the subcontractors and farmers were African Americans. These

Page 31: 100+ BONUS ENGLISH QUESTIONS FROM ACT DIRECTIONS · 1 Bonus questions to accompany The Official ACT Prep Guide, 2016‐2017 (ISBN 9781119225416), published by Wiley. 100+ BONUS ENGLISH

31 

Bonus questions to accompany The Official ACT Prep Guide, 2016‐2017 (ISBN 9781119225416), published by Wiley. 

phrases clarify the writer's point that Glory Foods employs African Americans "whenever possible."

The best answer is NOT:

G because even without the phrases, Paragraph 4 clearly explains Glory Foods' attempt to employ African American contractors.

H because the phrases "whenever possible" and "much of" are not examples of wordiness; rather, they clearly inform the reader.

J because the phrases do not describe the subcontractors or farmers yet they are essential to the meaning of the sentence.

Question 41. The best answer is A because it correctly uses the infinitive form of the verb (to evoke) after the verb of intention (is meant).

The best answer is NOT:

B because the verb phrase "is meant at evoking" is not an idiom of standard written English and confuses the reader.

C because it is incorrect in a way that is similar to the problem in B. The verb phrase "is meant in evoking of" is not idiomatic English and results in an unclear statement.

D because omitting the infinitive to evoke also results in a phrase that is not standard written English. The verb is meant needs to be followed by an infinitive verb form-in this case to evoke.

Question 42. The best answer is J because it is the clearest, most logical statement. The prepositional phrase "during the Civil War" clearly modifies "a black regiment." Modifying phrases should be placed as near as possible to the words they modify, which is why "during the Civil War" is best placed at the end of this sentence.

The best answer is NOT:

F because the phrase "during the Civil War" appears to modify "the movie." This placement wrongly suggests that the movie was filmed and shown during the Civil War. In addition, the phrase "of the same name" appears to modify "the Civil War" instead of "the movie."

G because the phrase "during the Civil War" appears to modify "the same name," which makes no sense.

H because the phrase "during the Civil War" appears to modify "of a black regiment." Again, the resulting statement "which tells the story during the Civil War of a black regiment" reads as though the movie was shown during the war.

Question 43. The best answer is C because it clearly explains why the writer should not add the information about the actor who starred in the film Glory. This information is not in keeping with the main point of the paragraph, which is to explain how the company got its name. Adding information about an actor distracts the reader from the focus of the paragraph and the essay as a whole.

The best answer is NOT:

A because it suggests that the sentence belongs in the paragraph when it clearly does not. Information about the actor who starred in Glory is not relevant at this point in the essay.

Page 32: 100+ BONUS ENGLISH QUESTIONS FROM ACT DIRECTIONS · 1 Bonus questions to accompany The Official ACT Prep Guide, 2016‐2017 (ISBN 9781119225416), published by Wiley. 100+ BONUS ENGLISH

32 

Bonus questions to accompany The Official ACT Prep Guide, 2016‐2017 (ISBN 9781119225416), published by Wiley. 

B because it, too, wrongly suggests that the sentence belongs in the paragraph.

D because even though it does indicate that the writer should not add the sentence, the reason given for not making this addition makes no sense. Including additional information saying that Bill Williams had met the actor Denzel Washington would also be irrelevant to the essay.

Question 44. The best answer is J because it maintains the present tense (are). Notice that present tense is used throughout the essay. A tense shift here would be illogical.

The best answer is NOT:

F because it makes an illogical and confusing shift from present tense to past tense.

G because it makes an illogical and confusing shift from present tense to past perfect tense.

H because it makes an illogical and confusing shift from the present tense to the past conditional.

Question 45. The best answer is A because it provides the most effective introductory paragraph. This is the best opening for the essay because it introduces the main topic, which is Bill Williams and his company, Glory Foods.

The best answer is NOT:

B because Paragraph 2 would be an ineffective and confusing opening for this essay. Look at its first sentence: "Even as a child, Williams loved to prepare food." The clue that this is not a good opening sentence is that most essays would not begin this abruptly. The reader would not know who this Williams person was.

C because it has the same problem that B does. Placing Paragraph 1 after Paragraph 3 makes Paragraph 2 the opening paragraph, but that paragraph begins too abruptly to provide an effective introduction.

D because it is incorrect for the same basic reason that B and C are.

PASSAGE IV: Pinball and Chance

Question 46. The best answer is J because the paragraph is more focused when the underlined portion is omitted. Mentioning the writer's trip to the movies diverts the reader's attention from the focus of the paragraph, which is a description of the game arcade.

The best answer is NOT:

F because adding information about the writer's trip to the movies is irrelevant to this paragraph and should be omitted. If you read the entire paragraph, you will see that this information does not belong.

G because it is incorrect in the same way that F is. It adds information that distracts the reader from the main focus of this introductory paragraph.

H because it is incorrect in the same way that F and G are. Even though this information is set off by parentheses, it still distracts the reader and is irrelevant.

Question 47. The best answer is B because the past tense (was) is consistent with the rest of the paragraph. In addition, the singular verb was is in agreement with the singular subject one.

The best answer is NOT:

Page 33: 100+ BONUS ENGLISH QUESTIONS FROM ACT DIRECTIONS · 1 Bonus questions to accompany The Official ACT Prep Guide, 2016‐2017 (ISBN 9781119225416), published by Wiley. 100+ BONUS ENGLISH

33 

Bonus questions to accompany The Official ACT Prep Guide, 2016‐2017 (ISBN 9781119225416), published by Wiley. 

A because the subject and verb do not agree in number. The subject one is singular and therefore requires a singular verb. The verb were is plural.

C because, again, the subject and verb do not agree. The subject one is singular, and the verb were is plural and therefore incorrect.

D because the past conditional tense (would have been) is inappropriate and confusing. In addition, adding the adjective actual would make the sentence unnecessarily wordy.

Question 48. The best answer is H because it provides the predicate are, which produces a complete sentence. A statement that has no predicate is a sentence fragment (an incomplete sentence).

The best answer is NOT:

F because placing the relative pronoun which between the subject ("Video screens") and predicate ("have been populated") creates a sentence fragment.

G because it is incorrect in the same way that F is. In this case, the relative pronoun that is placed between the subject and predicate.

J because it fails to provide a predicate, which creates another incomplete sentence.

Question 49. The best answer is A because it provides the most logical sequence of sentences for this paragraph. Sentence 4 provides a necessary link between the description of the video games in Sentences 1 through 3 and the description of the pinball machines in Sentence 5. In Sentence 4, the phrase "on the other hand" signals that this sentence is going to provide a contrasting point of view. In this case, the writer contrasts video games and pinball machines.

The best answer is NOT:

B because if Sentence 4 were placed right after Sentence 1, the paragraph would be incoherent, illogical, and confusing. Placing Sentence 4 here would interrupt the description of the video games with a comment about pinball machines.

C because placing Sentence 4 after Sentence 5 would confuse readers. They would not understand that the phrase "Some machines" in Sentence 5 actually refers to pinball machines. Also, the transitional phrase "on the other hand" in Sentence 4 does not logically follow the information in Sentence 5.

D because omitting Sentence 4 would confuse readers. The transition that Sentence 4 provides is a necessary link between the description of the video games in Sentences 1 through 3 and the description of the pinball machines in Sentence 5.

Question 50. The best answer is J because information about the durability of video games is not relevant to the writer's argument in this paragraph. The main point of the paragraph is that video games are more predictable than pinball machines. Adding information about how video games are "built to last" or are "constructed durably" distracts the reader.

The best answer is NOT:

F because, as stated above, information about how the machines have been "built to last" diverts the reader from the main focus of the paragraph.

G because it is incorrect in the same way that F is. Adding the irrelevant information that the machine "is constructed durably" is distracting to the reader.

Page 34: 100+ BONUS ENGLISH QUESTIONS FROM ACT DIRECTIONS · 1 Bonus questions to accompany The Official ACT Prep Guide, 2016‐2017 (ISBN 9781119225416), published by Wiley. 100+ BONUS ENGLISH

34 

Bonus questions to accompany The Official ACT Prep Guide, 2016‐2017 (ISBN 9781119225416), published by Wiley. 

H because it is incorrect in the same way that G and F are.

Question 51. The best answer is C because it maintains the second-person (You) perspective that is used throughout this paragraph. It is important to note that the writer is using the second-person point of view in this paragraph to speak directly to and draw in the reader. Consider the sentence preceding this one: "As you guide your character through the game's challenges, you come to know how the machine will respond to your every move."

The best answer is NOT:

A because it fails to maintain a consistent viewpoint. It makes an illogical shift from the second person (You) to the third person singular (He or she).

B because it makes an illogical shift from the second person (You) to the first person plural (We).

D because it not only shifts from second person (You) to third person (People), but it also illogically shifts from present tense (learn) to past tense (learned).

Question 52. The best answer is F. Notice that this question asks for the least acceptable answer. In other words, the best answer is the weakest choice. If you read the paragraph carefully, you will see that the idea presented in this sentence (pinball is unpredictable) is meant to contrast with the idea in the preceding sentence (video games are predictable). Given this context, using the transitional word therefore at this point is illogical and confusing.

The best answer is NOT:

G because it is an acceptable alternative to though, providing the same logical transition.

H because it also provides an acceptable alternative to though.

J because, along with G and H, it provides an acceptable alternative to though.

Question 53. The best answer is B because the absence of a comma here creates the clearest and most understandable sentence.

The best answer is NOT:

A because it places an unnecessary and distracting comma between the subject clause ("a ball you thought was lost") and the predicate ("will ... bounce").

C because it sets off the auxiliary (helping) verb will for no logical reason.

D because it places an unnecessary and distracting comma between the auxiliary verb will from the main verb bounce.

Question 54. The best answer is F because it most effectively links the topic of Paragraph 3

(pinball is less predictable than video games) and the topic of Paragraph 4 (the element of chance makes pinball more interesting than video games).

The best answer is NOT:

G because it fails to provide an effective link between the topics of the two paragraphs, as described above. This choice undermines the writer's argument by saying that pinball games are similar to video games.

H because it, too, fails to provide an effective transition from Paragraph 3 to Paragraph 4. H also contradicts the writer's previously stated point that pinball is challenging.

Page 35: 100+ BONUS ENGLISH QUESTIONS FROM ACT DIRECTIONS · 1 Bonus questions to accompany The Official ACT Prep Guide, 2016‐2017 (ISBN 9781119225416), published by Wiley. 100+ BONUS ENGLISH

35 

Bonus questions to accompany The Official ACT Prep Guide, 2016‐2017 (ISBN 9781119225416), published by Wiley. 

J because, like G and H, it provides an ineffective transition between the ideas presented in the two paragraphs. In addition, if you inserted this sentence at the beginning of Paragraph 4, the next sentence would not logically follow.

Question 55. The best answer is D because it results in the clearest and most concise response. In other words, it avoids redundancy (repeating the same idea) and wordiness.

The best answer is NOT:

A because it is redundant. At this point in the sentence, it is already clear that the writer is referring to "these video games."

B because it is incorrect for the same reason that A is.

C because it is incorrect in the same way that A and B are.

Question 56. The best answer is F because it maintains the second-person (you), present tense perspective that is used in the surrounding text. Notice that the preceding sentence establishes the second-person point of view: "Once you have mastered a game." (This question is similar to question 51.)

The best answer is NOT:

G because it makes a confusing tense shift in this sentence from the present perfect tense ("you have mastered") to the past tense ("you then looked").

H because it makes an illogical shift from the second-person plural (you) to the third person (one).

J because it, too, makes an illogical shift from the second-person plural (you) to the third person (one).

Question 57. The best answer is B because the comma between these two noun phrases ("sometimes your enemy" and "sometimes your ally") provides clarity for this sentence.

The best answer is NOT:

A because without the comma there, the statement becomes ambiguous and confusing. It's hard to tell whether the second sometimes is modifying your enemy or your ally.

C because it improperly uses a semicolon between these two noun phrases. By the way, those phrases are called "predicate nouns" because they follow the linking verb is.

D because even though the conjunction and could be used between these two sentence elements, setting off the conjunction with commas is inappropriate and confusing.

Question 58. The best answer is G because it provides the most concise way to make the writer's point.

The best answer is NOT:

F because it is vague and unnecessarily wordy. In addition, it creates a clause that lacks subject-verb agreement. The subject times is plural and requires a plural verb, not the singular verb is.

H because it is redundant (it repeats the same idea). In this sentence, each, single, and unique all mean the same thing.

J because the phrases "every single time" and "each" make this sentence wordy and repetitive.

Page 36: 100+ BONUS ENGLISH QUESTIONS FROM ACT DIRECTIONS · 1 Bonus questions to accompany The Official ACT Prep Guide, 2016‐2017 (ISBN 9781119225416), published by Wiley. 100+ BONUS ENGLISH

36 

Bonus questions to accompany The Official ACT Prep Guide, 2016‐2017 (ISBN 9781119225416), published by Wiley. 

Question 59. The best answer is C because it provides an adjective (continual) for the noun that it precedes (challenge).

The best answer is NOT:

A because the adverb continually lacks a neighboring sentence element that it can modify (a verb or an adjective).

B because the adverb continuously faces the same problem of lacking something to modify.

D because the verb form continue is simply out of place here between the article a and the noun challenge.

Question 60. The best answer is J because throughout the essay, the writer suggests that "pinball is superior" by making the argument that pinball requires more skill and is more challenging than video games. It is reasonable to conclude, then, that this essay fulfilled the writer's goal.

The best answer is NOT:

F because the writer does suggest in this essay that video games "might seem more attractive than pinball," but this has nothing to do with the writer's goal of writing an essay that shows pinball as being superior to video games.

G because this choice can be ruled out for two reasons: first, the essay does fulfill the writer's goal, and second, the writer does not say that video games challenge the skills of the player.

H because this answer states that the essay does fulfill the writer's goal, but the reason given is not accurate. The writer never states that pinball games "are more visually attractive than video games."

PASSAGE V: When a Computer Gets Sick ...

Question 61. The best answer is D because the absence of commas here creates the clearest and most understandable sentence.

The best answer is NOT:

A because it inserts an unnecessary and confusing comma between the noun letters and the preposition in. The phrase "in front of you" modifies letters, and these elements should not be separated by any punctuation.

B because it inappropriately and confusingly inserts a comma between the verb tumbles and the preposition to.

C because it inserts an unnecessary and confusing comma between the complete subject phrase ("each of the letters in front of you") and the verb tumbles.

Question 62. The best answer is H because it effectively and logically uses the coordinating conjunction (linking word) and to connect the two independent clauses in this sentence.

The best answer is NOT:

F because the use of the conjunction unless here creates an illogical statement. It makes no sense to say, "The computer is probably sick, unless ... [it] has a virus."

Page 37: 100+ BONUS ENGLISH QUESTIONS FROM ACT DIRECTIONS · 1 Bonus questions to accompany The Official ACT Prep Guide, 2016‐2017 (ISBN 9781119225416), published by Wiley. 100+ BONUS ENGLISH

37 

Bonus questions to accompany The Official ACT Prep Guide, 2016‐2017 (ISBN 9781119225416), published by Wiley. 

G because the use of the conjunction except also creates an illogical statement. A computer with a virus is not an exception to a computer being sick.

J because the use of the conjunction as if creates a confusing and ambiguous sentence.

Question 63. The best answer is D because it appropriately sets off the conjunctive adverb however with commas. When a conjunctive adverb or transitional expression interrupts a clause, as however does in this sentence, it should usually be set off with commas.

The best answer is NOT:

A because it places a comma after however but omits the corresponding comma before the word.

B because it inappropriately uses a semicolon instead of a comma before the adverb however.

C because it is incorrect because it places a comma before however but omits the corresponding comma after the word.

Question 64. The best answer is G because it provides the most effective introductory sentence for Paragraph 3. The topic of the paragraph is the serious computer viruses known as "bombs." Notice that the second sentence in this paragraph logically follows and further defines these "bombs."

The best answer is NOT:

F because the information is too broad. The topic of the paragraph is not all computer viruses; rather, it is the much narrower topic of viruses called "bombs." In addition, if this sentence opened the paragraph, the reader would not understand, later in the paragraph, what "these bombs" referred to.

H because it introduces a topic other than the "bombs." If you read the paragraph with this sentence as the introduction, the paragraph makes no sense.

J because it, too, strays from the main topic of the paragraph, which is the computer viruses referred to as "bombs."

Question 65. The best answer is A because it most clearly and concisely expresses the point being made in this sentence.

The best answer is NOT:

B because the phrase "devastative disaster" is redundant (repeating information). In this case, the use of the adjective devastative is redundant because the noun disaster expresses the same thought.

C because this phrasing is confusing if not nonsensical. How can something be "devastation to the operating of a computer"?

D because it is redundant and unclear. The redundancy occurs in the phrase "can be possibly." Also, the phrase "operating of a computer" is not idiomatic English; that is, it's not the way English speakers would normally say or write the phrase.

Question 66. The best answer is F because it is the clearest and most concise of the four choices given.

The best answer is NOT:

Page 38: 100+ BONUS ENGLISH QUESTIONS FROM ACT DIRECTIONS · 1 Bonus questions to accompany The Official ACT Prep Guide, 2016‐2017 (ISBN 9781119225416), published by Wiley. 100+ BONUS ENGLISH

38 

Bonus questions to accompany The Official ACT Prep Guide, 2016‐2017 (ISBN 9781119225416), published by Wiley. 

G because the phrase "that detect computer viruses" is redundant. The phrase is not necessary because the reader already knows that detection programs, by definition, "detect computer viruses." In addition, the phrase "computer viruses" appears a second time at the end of the sentence.

H because it is also unnecessarily wordy and redundant. The phrase "computer viruses" appears twice in this short sentence.

J because it, too, unnecessarily repeats the phrase "computer viruses."

Question 67. The best answer is D because both verbs in the subordinate, or dependent, clause of this sentence agree in number with the subject programs, which the clause modifies. The subject programs is plural, so the verbs search and destroy must also be plural.

The best answer is NOT:

A because the singular verb forms (searches, destroys) do not agree with their plural subject, programs.

B because it is incorrect in the same way that A is; the singular verb forms do not agree with their plural subject.

C because it is ungrammatical. Although the verb search is plural and agrees with the subject programs, the verb destroys is singular and does not agree.

Question 68. The best answer is G because it provides a predicate verb (is found) for the main clause of this sentence. ("Evidence ... is found in the names of their programs.")

The best answer is NOT:

F because it creates an incomplete sentence, a fragment. The participle being found cannot function as the predicate verb of the main clause of this sentence.

H because, like F, it creates a fragment because the participle having been found cannot function as a predicate verb.

J because, in this case, the verb form found reads as if it were a past participle, not a predicate verb. This too is a sentence fragment because it lacks a predicate for the sentence's main clause.

Question 69. The best answer is A because it appropriately uses a colon to introduce the list of names of the programs. Introducing a list is one function of the colon. .

The best answer is NOT:

B because it improperly uses the semicolon, which is generally used to separate two independent clauses.

C because it omits the necessary punctuation. The colon is needed here to signal to readers that a list of "the names of their programs" will follow.

D because the comma is not a "strong" enough punctuation mark here. Is this comma the same as or part of the series of commas that follow in this sentence?

Question 70. The best answer is F because it is the clearest and most logical of the four choices. It also provides a proper idiom of standard written English. The phrase "As with all diseases" indicates that the best cure for computer viruses is that same as that for all diseases: prevention.

Page 39: 100+ BONUS ENGLISH QUESTIONS FROM ACT DIRECTIONS · 1 Bonus questions to accompany The Official ACT Prep Guide, 2016‐2017 (ISBN 9781119225416), published by Wiley. 100+ BONUS ENGLISH

39 

Bonus questions to accompany The Official ACT Prep Guide, 2016‐2017 (ISBN 9781119225416), published by Wiley. 

The best answer is NOT:

G because it results in an ambiguous and illogical statement. It states that "the best cure" is "similarly to all diseases," which is illogical and ungrammatical.

H because it is unclear. The use of the phrase "In the same way as" suggests that the writer is trying to compare apples ("all diseases") and oranges ("the best cure").

J because "According with" is not an idiom of standard written English. Even if the sentence began "According to all diseases," which is idiomatic, the sentence still wouldn't make sense.

Question 71. The best answer is C because the question states that the writer's intention is to recommend "ways to protect computer data against viruses." C provides a recommendation by advising the reader to use antivirus programs frequently.

The best answer is NOT:

A because stating that many "viruses are quite sophisticated" does not provide a recommendation and, thus, does not accomplish the writer's stated intention.

B because it does provide a recommendation of sorts, but not the recommendation stated in the question. Adding software that checks spelling does not "protect computer data against viruses."

D because although this choice makes a broad recommendation ("be aware of the various ways to prevent viruses"), it does not recommend "specific ways to protect to protect computer data," which is the writer's stated intention.

Question 72. The best answer is H because placing the word better into the phrasing "you had better hope" provides the clearest statement and best clarifies the meaning of the sentence.

The best answer is NOT:

F because here, better inappropriately modifies responds, which confuses the meaning of the sentence. Also, the resultant phrase "you had hope" sounds wrong in this sentence along with its two present tense verbs (is and responds).

G because in this arrangement, better modifies system and implies a comparison that does not exist. "If there is a virus in your better system" wrongly suggests that there are two systems.

J because it is incorrect for the same reason that F is.

Question 73. The best answer is C because the most logical and effective placement for this sentence is after Paragraph 4. The last sentence of this paragraph lists the names of the virus detection programs; the new sentence, which refers to those names, logically follows. Also, this new sentence states that the "Names ... suggest that the problem is serious." The names identified in Paragraph 4 (Vaccine, Checkup, Antitoxin, and Disinfectant) do imply a level of seriousness.

The best answer is NOT:

A because Paragraph 2 provides a description of what a computer virus can do and does not refer to any "names" of viruses. Adding the sentence here makes no sense.

B because the main topic of Paragraph 3 is computer "bombs." As with A, there is no reference to particular "names."

Page 40: 100+ BONUS ENGLISH QUESTIONS FROM ACT DIRECTIONS · 1 Bonus questions to accompany The Official ACT Prep Guide, 2016‐2017 (ISBN 9781119225416), published by Wiley. 100+ BONUS ENGLISH

40 

Bonus questions to accompany The Official ACT Prep Guide, 2016‐2017 (ISBN 9781119225416), published by Wiley. 

D because it, too, would be an illogical placement. Although there are "names" in the last sentence of Paragraph 5 (Internet and World Wide Web), these names do not suggest a serious problem in the way that the "names" in Paragraph 4 do.

Question 74. The best answer is H because the second-person pronouns (you, your) do directly address the reader. Revising the essay so that it used the third-person pronouns one and one's would sacrifice that sense of addressing and advising the reader.

The best answer is NOT:

F because, although shifting to the third-person pronouns one and one's may change the tone of the essay, a polite and formal tone is clearly not appropriate to the purpose of this piece.

G because changing the pronouns in the essay from second to third person would not suggest that the writer is "speaking to a broader and more inclusive audience."

J because, in this essay, the setting is not a prominent element, and a shift in pronouns from second to third person would not change the sense of that setting.

Question 75. The best answer is D because the essay does limit itself "to describing computer viruses and the basic precautions to be taken against them." The essay does not discuss the ethics of tampering with a computer system, so it would not meet the writer's goal as described in this question.

The best answer is NOT:

A because the essay does not explain any moral or ethical consequences resulting from computer viruses.

B because the process of ridding a computer system of viruses is not explained in detail.

C because a reader would not necessarily have to know how a virus is programmed in order to make a judgment about the morality or ethics of programming a virus.

PASSAGE VI: Grandpa's Remote Control

Question 76. The best answer is A because it provides punctuation (in this case, a period) that appropriately separates these two complete thoughts or statements – “My grandfather is not known for embracing technological change” and “He still drives his ’59 Chevy Impala.”

The best answer is NOT:

B because it creates a run-on, or fused sentence. There is no punctuation or conjunction (connect word) between the two statements.

C because the phrase “still driving his ’59 Chevy Impala” is not a complete statement (because there is no stated subject). It could work in this sentence if it were set off from the main clause with a comma. It would then modify “My grandfather,” the subject of the main clause. But this answer doesn’t provide that punctuation.

D because the conjunction and that connects the phrase “still driving his ’59 Chevy Impala” to the main clause creates confusion by linking groups of words that are not grammatically parallel.

Page 41: 100+ BONUS ENGLISH QUESTIONS FROM ACT DIRECTIONS · 1 Bonus questions to accompany The Official ACT Prep Guide, 2016‐2017 (ISBN 9781119225416), published by Wiley. 100+ BONUS ENGLISH

41 

Bonus questions to accompany The Official ACT Prep Guide, 2016‐2017 (ISBN 9781119225416), published by Wiley. 

Question 77. The best answer is G because it correctly punctuates this sentence, which is actually a fairly simple subject-verb-direct object sentence except that the direct object is a noun clause. The sentence could also be written with the word that introducing the clause.

The best answer is NOT:

F because it inserts an unnecessary comma between the verb says and the direct object, which is what he says (“he can’t imagine needing frivolous options like automatic transmission or power steering”). It’s worth pointing out that the comma would be correct if what followed the verb were a direct quotation, as in speech: He says, “I can’t imagine needing frivolous options.”

H because it inserts an unnecessary comma between the verb and the direct object clause.

J because it adds the same unnecessary comma as H does, plus it places an unnecessary comma between the pronoun that and the clause it introduces.

Question 78. The best answer is C because it provides the correct past tense form of the irregular verb go.

The best answer is NOT:

A because the verb has went is grammatically incorrect. It should be has gone (which would still be incorrect here because it would create an awkward tense shift).

B because the verb had went is grammatically incorrect. It should be had gone.

D because it is in the present tense, creating a shift in verb tense in this sentence. The three verbs that follow this one in the sentence are all in the past tense –had quit, tried, and resisted.

Question 79. The best answer is H because the coordinating word because clearly and concisely links this parenthetical clause (“because his old black-and-white model had finally quit”) to the preceding clause (“he went to buy a new color television”).

The best answer is NOT:

F because it is excessively wordy and clunky. The phrase “owing to the knowledge that his old black-and-white model had finally quit” doesn’t express anything that the clause “because his old black-and-white model had finally quit” doesn’t express more clearly and more precisely.

G because it is unnecessarily wordy. The clause “due to the understandable fact that his old black-and-white model had finally quit” seems, on first glance, impressive. But it is empty, pretentious language, and not consistent with the style of the rest of this essay.

J because the coordinating word so does not logically link this parenthetical clause to the preceding clause.

Question 80. The best answer is B because it adds an appropriate and relevant detail here. In this sentence, the writer is expressing that the grandfather believed he was still healthy and had no need of a remote control for his television—“He said that he had two good legs and was perfectly capable of getting out of his chair.” The addition of “when he wanted to change the channel” fits with the point of the rest of the sentence.

The best answer is NOT:

A because the fact that the chair “was made of black leather” is irrelevant to the writer’s purpose here.

Page 42: 100+ BONUS ENGLISH QUESTIONS FROM ACT DIRECTIONS · 1 Bonus questions to accompany The Official ACT Prep Guide, 2016‐2017 (ISBN 9781119225416), published by Wiley. 100+ BONUS ENGLISH

42 

Bonus questions to accompany The Official ACT Prep Guide, 2016‐2017 (ISBN 9781119225416), published by Wiley. 

C because the fact that the chair is located “by the south window in the family room” is a pointless digression in this sentence.

D because, again, the fact that the grandfather “liked to sit” in his chair is an insignificant detail here. Since it was “his chair,” he would presumably like to sit there.

Question 81. The best answer is J because, here, no adverb or phrase is needed to make a connection between these two sentences. The essay works fine here without that kind of help.

The best answer is NOT:

F because the adverb however, as it is used here, indicates that this sentence is going to contradict or contrast with the statement in the preceding sentence. This sentence does not do that.

G because the adverb additionally indicates that this sentence is going to add a point that builds on the statement in the preceding sentence. This sentence does not do that

H because the adverb conversely expresses the same general idea that however does. This sentence does not provide a contrast to the statement in the preceding sentence.

Question 82. The best answer is C because the absence of commas here creates the clearest and most understandable sentence. This is another case (like question 6) where less is more.

The best answer is NOT:

A because it inserts a comma that confounds your ability to understand the sentence. The comma after the word time makes you think that Grandpa punched in the time and then he punched in the channel of his favorite news program, when actually he punched in the time that the program came on and the channel it was on.

B because it also inserts an unnecessary and confusing comma, this time after the conjunction and rather than just before it.

D because the comma after the word channel indicates that the phrase "of his favorite daily news program" is not essential. It's set off from the rest of the sentence with commas, as if it were parenthetical—a nice, piece of information but not necessary to the meaning of the sentence. But if you try reading the sentence without this phrase, the meaning of the sentence is no longer clear.

Question 83. The best answer is J because the phrase On the other hand signals that this sentence is going to provide a contrasting point or a different perspective. Grandpa's logic goes like this: The remote control's volume button is great for drowning out someone he doesn't want to hear; on the other hand (from another perspective), the mute button is great for silencing the television when he doesn't want to hear it.

The best answer is NOT:

F because the phrase For example indicates that this sentence will offer an example of the statement expressed in the sentence before it. This sentence does not provide that payoff.

G because it is incorrect for the same reason that F is. The phrase To illustrate is fairly close in meaning to For example.

H because the phrase On the one hand suggests that the writer is going to make a point here and then make another point. But the other point never gets made here; this is the last sentence in this paragraph.

Page 43: 100+ BONUS ENGLISH QUESTIONS FROM ACT DIRECTIONS · 1 Bonus questions to accompany The Official ACT Prep Guide, 2016‐2017 (ISBN 9781119225416), published by Wiley. 100+ BONUS ENGLISH

43 

Bonus questions to accompany The Official ACT Prep Guide, 2016‐2017 (ISBN 9781119225416), published by Wiley. 

Question 84. The best answer is B because advertisers is a specific noun that clearly communicates who Grandpa wants to be able to cut off in midsentence.

The best answer is NOT:

A because the pronoun them does not have a clear antecedent (a noun that it refers to and stands in for).

C because the pronoun it seems to refer back to the noun sound, but that is nonsensical. How do you "cut sound off in midsentence"?

D because the possessive pronoun its might refer back to the noun sound or to the noun button, but both of those would be nonsensical. What do you make of "the button that mutes the sound lets him cut the button's function off in midsentence"?

Question 85. The best answer is F because it appropriately punctuates this complex sentence. A complex sentence, by definition, contains an independent clause ("This option automatically turns the TV off after a preset amount of time") and one or more dependent clauses ("which is very convenient" and "when he falls asleep while watching a show").

The best answer is NOT:

G because by inserting a comma before the clause "when he falls asleep while watching a show" and setting it off from the rest of the sentence, the writer is signaling that the clause is not essential to the meaning of the rest of the sentence. But that is not so. Try reading the sentence without that final clause. The option of automatically turning off the TV is not always convenient, but it's sure handy when Grandpa falls asleep in the middle of a show.

H because it creates a sentence fragment. "When he falls asleep while watching a show" cannot stand alone. It's not a complete thought. Readers want to know more.

J because it is incorrect for the same reason that H is wrong. As it is used here, the semicolon is signaling that the statements on either side of the semicolon should be independent clauses (complete thoughts).

Question 86. The best answer is D. Think of this sentence as a jigsaw puzzle. Each puzzle piece is a phrase or clause. In this version, all the pieces of this sentence fit together well. And the "picture" that results looks like something that we can understand.

The best answer is NOT:

A because the parts of this complex sentence are poorly arranged—to the point of nonsense. Perhaps the best way of answering this question is to listen carefully to each wording as you read it out loud to yourself. The clause "what he wants his TV doing ... is to know a source of both pleasure and power" is pretty funny, but it's probably not what the writer meant to say here.

B because of a similarly flawed arrangement of the elements of this sentence. Placing the introductory clause "Even when he sleeps" directly before the statement "Grandpa says" gives readers the wrong impression that Grandpa is talking in his sleep.

C because of its clumsiness. The following makes little or no sense at all: "Doing what he wants . . . is to know his TV is a source of both pleasure and power for him."

Question 87. The best answer is F because it provides a main clause for this sentence: "Grandpa points his finger at the TV and ... seemingly turns it on." Notice that the phrase

Page 44: 100+ BONUS ENGLISH QUESTIONS FROM ACT DIRECTIONS · 1 Bonus questions to accompany The Official ACT Prep Guide, 2016‐2017 (ISBN 9781119225416), published by Wiley. 100+ BONUS ENGLISH

44 

Bonus questions to accompany The Official ACT Prep Guide, 2016‐2017 (ISBN 9781119225416), published by Wiley. 

introducing the sentence (which is often called a participial phrase) cannot be a main clause because it has no stated subject.

The best answer is NOT:

G because it creates a sentence fragment. There is no stated subject and no main clause.

H because it is incorrect for the same reason that G is incorrect—no subject, no main clause.

J not because of a fragment problem but because of a tense shift problem. Both the verb in the preceding sentence (uses) and the second verb in the compound predicate in this sentence (turns) express action in the present. It doesn't make sense to place a verb here (has pointed) that expresses action that began in the past and continues in the present.

Question 88. The best answer is A because it provides a fitting ending for this personal essay because it refers to Grandpa's continuing discovery of creative ways of using his new television remote control.

The best answer is NOT:

B because it provides a description of Grandpa as uninterested in learning how to use his remote control. That's inconsistent with the rest of the essay's portrayal of him as embracing this new technology.

C because it introduces elements that are insignificant in terms of the rest of the essay. The 200-page manual had not been mentioned elsewhere in the story, and the salesperson had only a minor role early in the story.

D because it suggests that the grandmother's feelings or thoughts are central to the essay, but the rest of the essay has focused on the actions and opinions of the grandfather.

Question 89. The best answer is G because the placement that creates the most logical order is between Sentences 1 and 2. First, the statement that Grandpa likes to use the programming function to play jokes on his grandchildren. Then, the explanation of how he does it: He programs the TV to turn on at a certain time; in the grandchildren's presence, he points his finger at the TV when it's programmed to turn on.

The best answer is NOT:

F because it is not the most logical place to add this information. It doesn't make sense to explain Grandpa's TV programming trick before the reader even knows that Grandpa likes to use the programming function to play jokes on his grandchildren.

H because it reverses the chronological order of how Grandpa's joke works. He can't play the trick on the grandchildren and then program the TV to turn on at a certain time.

J because it is incorrect for the same reason that H is. The chronological order of the setup of the remote control-finger joke is out of whack.

Question 90. The best answer is D because it provides an effective transition from the first part of the essay to the second part. The opening paragraphs focus on Grandpa's resistance to but eventual acquisition of a new TV with a remote control. The last sentence of Paragraph 2 indicates that Grandpa overcame his resistance to technology and began to use the remote. The first sentence of Paragraph 3 points· out that Grandpa has actually grown interested in the. uses

Page 45: 100+ BONUS ENGLISH QUESTIONS FROM ACT DIRECTIONS · 1 Bonus questions to accompany The Official ACT Prep Guide, 2016‐2017 (ISBN 9781119225416), published by Wiley. 100+ BONUS ENGLISH

45 

Bonus questions to accompany The Official ACT Prep Guide, 2016‐2017 (ISBN 9781119225416), published by Wiley. 

of the remote, and this leads into the rest of the essay's description of the uses that he discovers, many of which wouldn't be found in that 200-page manual.

The best answer is NOT:

A because the first sentence of Paragraph 3 is "Grandpa is intrigued by the various uses for that remote." This sentence suggests that the remote control might have intriguing uses, but it doesn't provide any information about those uses. '

B because, again, the sentence does suggest that Grandpa liked using the remote, but it doesn't provide any details that support that as a fact.

C because it would be a vast overstatement to say that this sentence is "a humorous blend of descriptive details and relevant information."

PASSAGE VII: "Topping Out" the Gateway Arch

Question 91. The best answer is A because it provides the punctuation (a comma) that best indicates the relationship between the introductory prepositional phrases ("During the early morning hours of October 28, 1965") and the main clause ("engineers stationed 630 feet above the ground made careful measurements").

The best answer is NOT:

B because the connecting word and creates confusion here. A word like and usually connects similar kinds of grammatical units-nouns (Tom and Mary), verbs (wander and search), adverbs (high and low), etc. Here, the word tries to connect an introductory phrase and a main clause. ·

C because placing a period here creates a sentence fragment: "During the early morning hours of October 28, 1965."

D because it is incorrect for the same reason that C is wrong. A semicolon is normally used to connect two independent clauses (that is, clauses that could each stand alone as complete sentences).

Question 92. The best answer is H because it shows the correct use of the apostrophe in expressions of time. You're probably more familiar with the uses of the apostrophe to express possession or ownership or to indicate a contraction (letters left out when words are combined). But you've probably seen phrases such as "yesterday's news" and "tomorrow's headlines" and "an hour's delay" —they all follow the same punctuation rule as "the day's work."

The best answer is NOT:

F because it leaves out that necessary apostrophe. Without the apostrophe, a reader might misread "the days" as a subject and "work" as a predicate. ·

G because it places the apostrophe in the wrong location. The rest of this sentence makes it clear that this is one day's work.

J because one would never use an apostrophe like this—by adding 's to the end of a plural noun.

Page 46: 100+ BONUS ENGLISH QUESTIONS FROM ACT DIRECTIONS · 1 Bonus questions to accompany The Official ACT Prep Guide, 2016‐2017 (ISBN 9781119225416), published by Wiley. 100+ BONUS ENGLISH

46 

Bonus questions to accompany The Official ACT Prep Guide, 2016‐2017 (ISBN 9781119225416), published by Wiley. 

Question 93. The best answer is A because this story of the last step in the construction of the Gateway Arch is told in the past tense. The past tense verb threatened is consistent with the other predicate verbs used in this paragraph—stationed, made, indicated.

The best answer is NOT:

B because the passive voice of the verb had been threatened doesn't make sense in this sentence.

C because the future perfect tense verb will have threatened creates an awkward and confusing shift from the past tense used elsewhere in this essay.

D because the present tense verb threatens creates a similarly awkward and confusing shift in tense.

Question 94. The best answer is J because it provides the most concise wording for this sentence and avoids the pointless repetition of the other choices.

The best answer is NOT:

F because it is pointlessly repetitive. The word delay doesn't add any information here that's not already clearly expressed by the word postpone.

G because the phrase "postpone to a later time" is pointlessly wordy. Can you postpone something to an earlier time? The information "to a later time" is already clearly expressed by the word postpone.

H because it is incorrect for the same reason that F is. The phrase "postpone by delaying" is another example of verbal overkill. Readers can sometimes feel disrespected when an essay tells them the same thing over and over.

Question 95. The best answer is B because this sentence explains that the topping-out ceremony being referred to here is "the placement of the final section between the two freestanding legs of the St. Louis Gateway Arch." Without this phrase, we wouldn't be able to figure out what "topping out" meant until we were halfway into the essay.

The best answer is NOT:

A because the essay would not lose a minor detail if this phrase were deleted. This is a key piece of information.

C because the wording of this phrase is straightforward and factual—there's nothing to suggest that the writer is expressing his or her opinion on the significance of anything.

D because the phrase does not state the ceremony's importance to St. Louis residents. Readers might draw a conclusion about the ceremony's importance based on the essay as a whole, but that's something else entirely.

Question 96. The best answer is F because the phrase "Luther Ely Smith" is an appositive for (renames or explains) the phrase it follows, "attorney and civic leader." Most appositive phrases are set off by commas, but this one should not be, because it is essential to the meaning of the sentence. Try reading the sentence without "Luther Ely Smith"—doesn't it sound strange or clunky?

The best answer is NOT:

Page 47: 100+ BONUS ENGLISH QUESTIONS FROM ACT DIRECTIONS · 1 Bonus questions to accompany The Official ACT Prep Guide, 2016‐2017 (ISBN 9781119225416), published by Wiley. 100+ BONUS ENGLISH

47 

Bonus questions to accompany The Official ACT Prep Guide, 2016‐2017 (ISBN 9781119225416), published by Wiley. 

G because the comma between the nouns attorney and civic leader is unnecessary and distracting. The word and is linking up these two nouns; the comma just gets in the way of that.

H because there should not be a comma separating the noun phrase "attorney and civic leader" and the noun phrase that defines and specifies it, "Luther Ely Smith."

J because it is incorrect for the reasons given for G and Has well as the reason that the proliferation of commas just totally confuses things: "In 1933, attorney, and civic leader" starts looking like it might be a series of three items.

Question 97. The best answer is C because this sentence states, "In 1933, attorney and civic leader Luther Ely Smith envisioned a memorial that would recognize St. Louis's major role in the westward expansion of the United States." It does provide some helpful background about the history leading up to the construction of the Gateway Arch, and it's the only place in the essay where such background is provided.

The best answer is NOT:

A because the sentence does state that St. Louis played a role in the westward expansion, but it does not explain why St. Louis played that role.

B because the sentence does state that Smith envisioned a memorial, but it does not mention what the memorial that he envisioned might look like.

D because the sentence does provide some biographical facts about Smith—he was an attorney and a civic leader, and he had an idea about a memorial. However, this information is not particularly crucial. The information that is most relevant and meaningful to this paragraph is that a memorial to St. Louis's role in the U.S. westward expansion (that is, the Gateway Arch) was first envisioned in 1933.

Question 98. The best answer is H because it is the only choice that makes this a complete sentence.

The best answer is NOT:

F because it creates a sentence fragment—a noun phrase ("Architect Eero Saarinen") and a dependent clause that modifies the noun phrase ("who created the design that symbolized the memorial's theme").

G because it too creates a sentence fragment—that same noun phrase and an appositive phrase that renames the noun phrase ("creator of the design that symbolized the memorial's theme").

J because it too creates a sentence fragment—that same noun phrase and a participial phrase modifying the noun phrase ("creating the design that symbolized the memorial's theme").

Question 99. The best answer is D because this question asks you to decide which word would provide the most logical and effective transition from one sentence to another. The best decision here is to use no transitional adverb at all. The preceding sentence states that Saarinen designed the Gateway Arch. This sentence is the first of three that describe the details of that design.

The best answer is NOT:

A because the sense of Meanwhile is that this event takes place at the same time that the preceding event takes place. That sense doesn't work logically here.

Page 48: 100+ BONUS ENGLISH QUESTIONS FROM ACT DIRECTIONS · 1 Bonus questions to accompany The Official ACT Prep Guide, 2016‐2017 (ISBN 9781119225416), published by Wiley. 100+ BONUS ENGLISH

48 

Bonus questions to accompany The Official ACT Prep Guide, 2016‐2017 (ISBN 9781119225416), published by Wiley. 

B because the sense of Therefore is that this condition exists or event takes place as a result of the preceding condition or event. That sense would work here only if we'd already been told that Saarinen's designs always look like the description that follows.

C because the sense of However is that this condition exists or event i:akes place in contrast to the preceding condition or event. That sense would work only if we'd already been told that Saarinen's designs never look like the description that follows.

Question 100. The best answer is F. All of these word choices are similar in meaning, having something to do with "decreasing or reducing in size," but they are not interchangeable synonyms. The context of this sentence helps us to decide the best choice: "spraying the [metal and concrete] leg [of the Gateway Arch] with water to cool it would make it contract." Merriam-Webster's Collegiate Dictionary (11th ed.) supports the accuracy of this choice, stating that "CONTRACT applies to a drawing together of surfaces or particles or a reduction of area or length" (p. 271).

The best answer is NOT:

G because reduce has more of a sense of "bringing down or lowering in size or degree or intensity," which is not exactly the action being described in this sentence. It's a much less precise word choice than contract is.

H because decrease has more of a sense of "declining in size or number or amount," which is not exactly the action being described here. It's a much less precise word choice than contract is.

J because compress primarily means "to reduce in size by pressing or squeezing," which is not the action being described here.

Question 101. The best answer is C because this phrasing uses the correct forms of the possessive pronouns their and its: "in their attempt to reduce its expansion."

The best answer is NOT:

A because it uses the contraction they're (meaning "they are") where the possessive pronoun their is called for.

B because it uses the contraction they're and the contraction it's (meaning "it is") where the possessive pronouns their and its are called for.

D because it uses the contraction it's where the possessive pronoun its is called for.

Question 102. The best answer is G because it provides the most logical and fluent arrangement of the possible parts of this sentence: an introductory dependent clause ("as the crowd cheered") followed by a main clause with a compound predicate ("the final section was hoisted up and welded to the two legs of the arch").

The best answer is NOT:

F because this arrangement of the pieces of information provides us with the nonsensical image of the final section being hoisted up after it had been welded to the legs of the arch.

H because this arrangement of the pieces of information provides us with the absurd image of the crowd being welded to the legs of the arch.

J because this arrangement of the pieces of information provides us with the confusing image of the crowd either doing some of the welding or being welded to the legs of the arch.

Page 49: 100+ BONUS ENGLISH QUESTIONS FROM ACT DIRECTIONS · 1 Bonus questions to accompany The Official ACT Prep Guide, 2016‐2017 (ISBN 9781119225416), published by Wiley. 100+ BONUS ENGLISH

49 

Bonus questions to accompany The Official ACT Prep Guide, 2016‐2017 (ISBN 9781119225416), published by Wiley. 

Question 103. The best answer is D because it offers the most concise wording and avoids the redundancy of the other choices. (Have you ever heard of the Department of Redundancy Department? That might be a handy way to remember what the word redundant means.)

The best answer is NOT:

A because, since a decade is ten years and three decades are thirty years, the phrase "Over three decades and more than thirty years" is redundant.

B because its phrasing is redundant and pointless. "Over three decades" do amount to "more than thirty years," but there's no reason to inform readers of that here.

C because the parenthetical phrase "over thirty years" merely repeats the time span just reported in a different measure of time (decades).

Question 104. The best answer is G because this is the LEAST acceptable alternative to the underlined portion. The sentence with the underlined portion reads, "Over three decades of planning and building had come to a conclusion, and the tallest monument in the United States was now complete." Replacing the word conclusion with halt creates a meaning problem in the context of this sentence because "coming to a halt" expresses the idea of stopping or suspending before or without completion.

The best answer is NOT:

F because it provides an acceptable wording. The phrase "reached completion" works in the context of this sentence just as well as "come to a conclusion" does.

H because it provides an acceptable wording. The phrase "come to an end" works in this sentence just as well as "come to a conclusion" does.

J because it provides an acceptable wording. The word "ended" works in this sentence just as well as "come to a conclusion" does.

Question 105. The best answer is C because the title of this essay—"'Topping Out' the Gateway Arch"—is a fairly good summary of the piece and actually helps us to arrive at the answer. The essay does not do a good job of describing the entire process of designing and building the Gateway Arch because it focuses instead on telling the story of this one step in the process.

The best answer is NOT:

A because this essay does mention the materials used to make the exterior and interior structural supports, but that doesn't mean it has described the entire design and construction process.

B simply because it presents an inaccurate description of this essay. An essay that did do what B claims to do would indeed fulfill the intended goal.

D because it is incorrect for the same reason as B is—it inaccurately describes this essay. This essay devotes just a few sentences to the early stages in the development of the arch.